Você está na página 1de 83

1.

~

Item: 1 of 44 1111 PMark <J C>- jf 9-•
·w
2 I

Q.ld: 3880 ( Previous Ne><t Lab Values Notes Calculator


• 4
5 A new screening test was devised to detect pancreatic cancer at early stages using a serum marker
6 (CA19-9) of the disease. A study of this new test showed that its use prolongs the survival of patients with
7 pancreatic cancer by several months. The researchers concluded that use of the test improves the
8 prognosis of patients with pancreatic cancer. Which of the following is a potential problem with this
·[;J
• 10
conclusion?
r A. Observer's bias [4%]
11
12 r B. Measurement bias [4%]
13 .., r C. Lead-time bias [81 %]
14
15 r D. Confounding [5%]
16 r E. Ascertainment bias [5%]
17

·[m
• 19 Explanation: User ld:
. 20
Lead-time bias should always be considered while evaluating any screening test. This bias occurs when
•-~
22 there is an incorrect assumption or conclusion of prolonged apparent survival and better prognosis due to a
23 screening test. What actually happens is that detection of the disease was made at an earlier point in time.
24 but the disease course itself or the prognosis did not change. so the screened patients appeared to live longer
25 from the time of diagnosis to the time of death. (USMLE tip: think of lead-time bias when you see "a new
26 screening test" for poor prognosis diseases like lung or pancreatic cancer.)

·c:]
• 28
(Choices A, B and E) Observer's bias. measurement bias and ascertainment bias refer to misclassification
of an outcome and/or exposure (e.g .. labeling diseased subjects as non-diseased and vice versa) and are
29
related to the design of the study. The scenario described does not mention how the study was designed.
30
31 (Choice D) Although the results of the study could be potentially confounded. there is no information on how
32 potential confounders were treated during the design or analysis stage of this study.
33
34 Educational Objective:
35 Understand the concept of lead-time bias in screening tests. The typical example of lead-time bias is
36 prolongation of apparent survival in patients to whom a test is applied. without changing the prognosis of the
37 disease.
38
39 Time Spent 11 seconds Copyright© USMLEWorld ,LLC. Last updated [7/26/20 13]
40
Item: 2 of 44 11 PMark <:J 1>- jf 9-• 1.":~ I

Q.ld: 3998 ( Previous Ne><t Lab Values Notes Calculator

A study was conducted to establish the average level of total serum cholesterol in a group of patients with
6 acute coronary syndrome. The study results were reported separately for males and females. The mean
7 total cholesterol level was 230 mg/dl for males and 220 mg/dl for females. Which of the following is the best
8 statistical method to compare the average cholesterol level for males and females?

:[lJ ., r
r
A. Two-sample t test [58%]
B. Two-sample z test [7%]
11
12 r C. Analysis of variance (ANOVA) [8%]
13 r D. Chi-square test [23%]
14
15
r E. Meta-analysis [3%]
16
17 Explanation: User ld:
·liBl
·~ The two-sample t test is commonly employed to compare two means. Several statistical approaches can be
• 20 used. butthe basic requirements needed to perform this test are the two mean values. the sample variances.
·1211 and the sample size. The t statistic is then obtained to calculate the p value. If the 'p' value is less than 0.05 .
. LBJ the null hypothesis (that there is no difference between two groups) is rejected. and the two means are
23 assumed to be statistically different. If the 'p' value is large. the null hypothesis is retained.
24
25 (Choice B) The two-sample z test can also be used to compare two means. but population (not sample)
26 variances are employed in the calculations. Because population variances are not usually known. the test has
·1271 limited applicability.
·~
29 (Choice C) The AN OVA (i.e .. analysis of variance) is used to compare three or more means.
30
31 (Choice D) The chi-square test is appropriate for categorical data and proportions.
32
(Choice E) Meta-analysis is an epidemiologic method of pooling the data from several studies to do an
33
analysis having a relatively big statistical power.
34
35 Educational Objective:
36 The two-sample t test is a statistical method that is commonly employed to compare the means of two
37 groups of subjects.
38
39
Time Spent 1 seconds Copyright© USMLEWorld ,LLC. Last updated [7/7/20 10]
40
Item: 3 of 44 11 f>Mark -<:1 C>- jf ~· 1.~ I

Q.ld: 7690 ( Previous Ne><t Lab Values Notes Calculator

Item 1 of2
5
6 Coronary-Artery Bypass Surgery in Patients with Left A 67-year-old male with recently diagnosed coronary artery
7 Ventricular Dysfunction disease comes for follow-up. He was hospitalized for
8
congestive heart failure 4 weeks ago and underwent coronary
:[lJ Hypothesis:

Is coronary-artery bypass grafting (CABG) superior to optimal


angiography showing significant narrowing of left anterior
descending and right coronary arteries. His left ventricular
11
medical therapy alone in the treatment of patients with coronary ejection fraction is 30%. Currently, he can walk 3 blocks on a
12
artery disease and heart failure? flat surface limited by chest discomfort and shortness of
13
breath. The physician tries to optimize his medical regimen.
14
Methods: He asks whether surgery is 'the right option' for him. Based on
15
the study results. which of the following is the best statement?
16
Design: Randomized clinical trial
17
·liBl r A. Medical therapy comes with a risk of exacerbating
Blinding: none
heart failure [3%]
·~
• 20 Median follow-up: 56 months r B. Once you have surgery you might need more
·1211 long-term procedures [3%]
. LBJ Setting: Multicenter (99 medical centers in 22 countries)
., r C. Surgery carries a higher initial risk of dying [41%]
23
24 Patients: Patients with a left ventricular ejection fraction of 35% or r D. Surgery would not affect your overall long-term
25 less and coronary artery disease amenable to CABG. Patients with well-being [48%]
26 a stenotic lesion leading to loss of 50% or more of the diameter of r E. The benefits of surgery are limited to preventing a
the left main coronary artery and/or Canadian Cardiovascular
·1271 Society class Ill or IV angina while receiving medical therapy were
heart attack [5%]
·~ excluded from the study. (The Canadian Cardiovascular Society
29
30 angina classification ranges from class 0. which indicates no Explanation: User ld:
31 symptoms, to class IV, which indicates angina at any level of
32 physical exertion.) The study above is a randomized clinical trial that looks at the
33
benefit of CABG with medical therapy versus medical therapy
34 Intervention: CABG plus optimal medical therapy vs optimal medical alone for the management of significant coronary artery
35
therapy alone disease and heart failure (defined as less than 35% left
36 ventricular ejection fraction). The study's primary end point
Outcome measures: The primary outcome was the rate of death was overall mortality.
37
from any cause. Major secondary outcomes included the rates of
38
death from cardiovascular causes and of death from any cause or The results are reported as a hazard ratio. which is the ratio of
39
hospitalization for cardiovascular causes. ~ an event rate occurring. ~n the treatment group co';lpare~ to an
40
2 Item: 3 of 44
Q. ld: 7690
• \' Mark <j
Previous
t:>
NeKt
G
::::l'
Lab Values

.V
Notes
,...
... j
Calculator
I

5
Results: The results are reported as a hazard ratiO , wh1ch 1s the ratiO of
6 an event rate occurring in the treatment group compared to an
A total of 1212 patients were randomly assigned to medical therapy event rate occurring in the non-treatment group. A ratio less
7
alone (602 patients) or medical therapy plus CABG (610 patients). than 1 indicates that the treatment group had a significantly
8
By the end of the follow-up period. 100 patients in the lower event rate while values greater than 1 indicate that the
:[lJ medical-therapy group ( 17%) underwent CABG, and 555 patients in
the CABG group (91%) underwent CABG.
treatment group had a much higher event rate .
11
In this study, the hazard ratio for all cause mortality was 0.86,
12
Primary ou1come (CABG vs medical therapy alone) bu1 the p value was not statistically significant (not less than
13 Rate of death from any cause Hazards Ratio (HR) 0.86 (0.72-1.04)
14
om). This means that surgery overall did not improve
P= 0.12 all-cause mortality during the average follow-up period of 56
15
months. CABG was also associated with a higher mortality in
16 Secondary outcomes (CABG vs medical therapy alone) the first 30 days after randomization with a hazard ratio of 3.12,
17 Death from any cause within 30 days after randomization HR 3.1 2 with a statistically significant p value less than O.D1 . This is due
·f1sl (1.33-7.31) P=0.006 to the inherent immediate risk of surgery and should be
-~ Death from cardiovascular causes HR 0.81 (0.66- 1.00) P=0.05 explicitly conveyed to the patient All the other secondary
• 20 Death from any cause or hospitalization for heart failure HR 0.84 outcomes were favorable for the CABG group.
:I;~ I
23
(0.71-0.98) P=0.03
Death from any cause or hospitalization for cardiovascular causes
HR 0.74 (0.64-0.85) P<0.001
(Choice A) The hazard ratio for death from any cause or
hospitalization for heart failure was 0.84, which means that the
24 Death from any cause or hospitalization for any cause HR 0.81 surgery (in addition to medical therapy) likely improves heart
25 (0.71-0.93) P=0.003 failure symptoms. This should not be interpreted as medical
26 Death from any cause or revascularization HR 0.60 (0.51-0.71) therapy worsening heart failure since medical therapy was
:I ;~I
29
P<0.001

Conclusion:
used in both groups, and it was the addition of surgery that
improved it.
30 (Choice B) The hazard ratio for death from any cause or
31 In this randomized trial. there was no significant difference between revasculari zation was 0.60, which indicates that the surgery
32 medical therapy alone and medical therapy plus CABG with respect added to medical therapy likely leads to fewer revascularization
33 to the primary end point of death from any cause. procedures than medical therapy alone.
34
35
Funding Source: Supported by grants from the National Heart. Lung. (Choice D) The ha zard ratio for death from any cause or
36 and Blood Institute and by Abbott Laboratories. hospitalization for any cause was 0.81. which indicates that the
37 surgery added to medical therapy likely leads to fewer
Structured abstract is based orr. NEJM 2011 ;364(17) 1607-16.
38 hospitalizations with better long-term well being than medical
39 therapy alone.
40
Item: 3 of 44
Q.ld: 7690 (
!II PMark <J
Previous
t>-
Ne><t
jf
Lab Values

Notes
1.":~
Calculator

5 Results: used in both groups. and it was the addition of surgery that
6 improved it.
A total of 1212 patients were randomly assigned to medical therapy
7
alone (602 patients) or medical therapy plus CABG (61 0 patients). (Choice B) The hazard ratio for death from any cause or
8
By the end of the follow-up period. 1DO patients in the revascularization was 0.60. which indicates that the surgery
:[lJ medical-therapy group ( 17%) underwent CABG. and 555 patients in
the CABG group (91%) underwent CABG.
added to medical therapy likely leads to fewer revascularization
procedures than medical therapy alone.
11
12 Primary outcome (CABG vs medical therapy alone) (Choice D) The hazard ratio for death from any cause or
13 Rate of death from any cause Hazards Ratio (HR) 0.86 (0. 72-1 .04)
14
hospitalization for any cause was 0.81 . which indicates that the
P= 0.12 surgery added to medical therapy likely leads to fewer
15
hospitalizations with better long-term well being than medical
16 Secondary outcomes (CABG vs medical therapy alone) therapy alone.
17 Death from any cause within 30 days after randomization HR 3.12
·liBl (1.33-7 .31) P=0.006 (Choice E) All the secondary outcomes except 30-day
·~ Death from cardiovascular causes HR 0.81 (0 .66-1 .DO) P=O .05 mortality were favorable for surgery with medical therapy,
• 20 Death from any cause or hospitalization for heart failure HR 0.84 indicating that there are other benefits to surgery with medical
·1211 (0. 71-0 .98) P=O .03 therapy other than just preventing heart attacks.
. LBJ Death from any cause or hospitalization for cardiovascular causes
23 HR 0. 74 (0 .64-0 .85) P<O .DO 1 Educational objective:
24 Death from any cause or hospitalization for any cause HR 0.81 Hazard ratios are the ratio of an event rate occurring in the
25 (0. 71-0 .93) P=O .003 treatment arm versus the non-treatment arm. Ratios less than
26 Death from any cause or revascularization HR 0.60 (0 .51-0.71) 1 indicate that the treatment arm had a lower event rate while
·1271 P<0.001 ratios higher than one indicate the treatment arm had a higher
·~ rate of events.
29 Conclusion:
30
References:
31 In this randomized trial, there was no significant difference between
32 medical therapy alone and medical therapy plus CABG with respect 1. Biostatistics primer: what a clinician ought to
33 to the primary end point of death from any cause. know: hazard ratios.
34 2. Estimation of the 2 -sample hazard ratio
35 Funding Source: Supported by grants from the National Heart, Lung.
and Blood Institute and by Abbott Laboratories. function using a semi parametric model.
36
37
Structured abstract is based on NEJM 2011; 364( 17) 1607-16.
38
Time Spent 23 Copyright© Last updated:
39
seconds USMLEWorld ,LLC. [7/1/2013]
40
Item: 4 of 44
Q.ld: 7691 (
11 PMark <:1
Previous
C>-
Ne><t
jf
Lab Values
""•
Notes
1.":~
Calculator
I

ltem2 of2
6 Coronary-Artery Bypass Surgery in Patients with Left As reported by investigators. by the end of the follow-up period.
7 Ventricular Dysfunction 17% of patients in the medical-therapy group underwent CABG.
8
and 91% of patients in the CABG group underwent CABG.
:[lJ Hypothesis:

Is coronary-artery bypass grafting (CABG) superior to optimal


Analyzing outcomes for patients from both groups who actually
underwent CABG versus received only medical therapy might
11
medical therapy alone in the treatment of patients with coronary introduce which of the following?
12
13 artery disease and heart failure?
14 r A. Lead-time bias [17%]
Methods:
15 r B. Measurement bias [34%]
16
Design: Randomized clinical trial r C. Observer bias [13%]
17
r D. Recall bias [4%]
·liBl Blinding: none
·~ .., r E. Susceptibility bias [32%]
• 20 Median follow-up: 56 months
·1211
. LBJ Setting: Multicenter (99 medical centers in 22 countries) Explanation: User ld:
23
24 Patients: Patients with a left ventricular ejection fraction of 35% or This study was a randomized clinical trial, but many patients in
25 less and coronary artery disease amenable to CABG. Patients with the medical therapy only arm also underwent surgery. By
26 a stenotic lesion leading to loss of 50% or more of the diameter of combining all surgical from both groups in one group and then
the left main coronary artery and/or Canadian Cardiovascular comparing against medical therapy may introduce susceptibility
·1271 Society class Ill or IV angina while receiving medical therapy were bias. which is a subgroup of selection bias that occurs when
·~ excluded from the study. (The Canadian Cardiovascular Society the treatment regimen selected for a patient depends on the
29
30 angina classification ranges from class 0. which indicates no severity of the patient's condition. This type of bias fails to take
31 symptoms, to class IV, which indicates angina at any level of into account other confounding variables that may be
32 physical exertion.) accounting for the patient's condition. This bias also negates
33 the benefits of randomization. which usually avoids selection
34 Intervention: CABG plus optimal medical therapy vs optimal medical bias and confounding variables.
35
therapy alone
In this study, 17% of the patients in the medical therapy group
36 Outcome measures: The primary outcome was the rate of death were not adequately treated with medical therapy and were
37
from any cause. Major secondary outcomes included the rates of selected to have CABG. There may have been other
38
death from cardiovascular causes and of death from any cause or confounding variables that caused this group of patients to fail
39
hospitalization for cardiovascular causes. ~ medical t~erapy that were not anal~zed in this study. ~y
40
Item: 4 of 44
Q.ld: 7691 (
!il P' Mark <:J
Previous
C>-
Ne><t
jf
Lab Values

Notes
1.":~
Calculator

In this study, 17% of the patients in the medical therapy group


were not adequately treated with medical therapy and were
6 Coronary-Artery Bypass Surgery in Patients with Left selected to have CABG. There may have been other
7 Ventricular Dysfunction confounding variables that caused this group of patients to fail
8
medical therapy that were not analyzed in this study. By
:[lJ Hypothesis:

Is coronary-artery bypass grafting (CABG) superior to optimal


grouping them with the patients in the surgery group. this may
lead to erroneous analysis and introduce more confounding
11
medical therapy alone in the treatment of patients with coronary factors. To avoid this in studies. patients are randomized and
12
artery disease and heart failure? analyzed separately to account for these possible confounding
13
variables.
14
15
Methods:
(Choice A) Lead time bias happens when two interventions
16 are compared to diagnose a disease. and one intervention
Design: Randomized clinical trial
17 diagnoses the disease earlier than the other without an effect
·liBl Blinding: none on the outcome (such as survival). This would make it appear
·~
• 20 Median follow-up: 56 months
that the intervention prolonged survival when it really just
diagnosed the disease sooner.
r
·1211
. LBJ Setting: Multicenter (99 medical centers in 22 countries) (Choice B) Measurement bias occurs from poor data
23 collection with inaccurate results. which is not what is
24 Patients: Patients with a left ventricular ejection fraction of 35% or described in this case.
25 less and coronary artery disease amenable to CABG. Patients with
26 a stenotic lesion leading to loss of 50% or more of the diameter of (Choice C) Observer bias occurs when the observer may be
the left main coronary artery and/or Canadian Cardiovascular
·1271 Society class Ill or IV angina while receiving medical therapy were
influenced by prior knowledge or details of the study that can
·~ excluded from the study. (The Canadian Cardiovascular Society
affectthe results. Blinded studies usually avoid this bias by
29 preventing the observers from knowing which treatment or
30 angina classification ranges from class 0. which indicates no intervention the participants are receiving. and also measuring
31 symptoms, to class IV, which indicates angina at any level of objective outcomes (such as mortality).
32 physical exertion.)
33 (Choice D) Recall bias occurs when a study participant is
34 Intervention: CABG plus optimal medical therapy vs optimal medical affected by prior knowledge to answer a question. This is more
35
therapy alone common in case-control studies rather than randomized
36 clinical trials.
Outcome measures: The primary outcome was the rate of death
37
from any cause. Major secondary outcomes included the rates of Educational objective:
38
death from cardiovascular causes and of death from any cause or Susceptibility bias is a type of selection bias where a treatment
39
hospitalization for cardiovascular causes. ~ regimen is selected for a patient based on the severity of their
40
Item: 4 of 44 1111 f' Mark <J [:>- jf ~· 1.":~ I

Q.ld: 7691 ( Previous Ne><t Lab Values Notes Calculator

(Choice A) Lead time bias happens when two interventions


are compared to diagnose a disease. and one intervention
6 Coronary-Artery Bypass Surgery in Patients with Left diagnoses the disease earlier than the other without an effect
7 Ventricular Dysfunction on the outcome (such as survival). This would make it appear
8 that the intervention prolonged survival when it really just
:[lJ Hypothesis:

Is coronary-artery bypass grafting (CABG) superior to optimal


diagnosed the disease sooner.

11 (Choice B) Measurement bias occurs from poor data


12 medical therapy alone in the treatment of patients with coronary collection with inaccurate results. which is not what is
13 artery disease and heart failure? described in this case.
14
15
Methods: (Choice C) Observer bias occurs when the observer may be
16 influenced by prior knowledge or details of the study that can
Design: Randomized clinical trial
17 affectthe results. Blinded studies usually avoid this bias by
·liBl Blinding: none preventing the observers from knowing which treatment or
·~ intervention the participants are receiving. and also measuring
• 20 Median follow-up: 56 months objective outcomes (such as mortality).
·1211
. LBJ Setting: Multicenter (99 medical centers in 22 countries) (Choice D) Recall bias occurs when a study participant is
23 affected by prior knowledge to answer a question. This is more
24
25
Patients: Patients with a left ventricular ejection fraction of 35% or
less and coronary artery disease amenable to CABG. Patients with
common in case-control studies rather than randomized
clinical trials.
r
26 a stenotic lesion leading to loss of 50% or more of the diameter of
the left main coronary artery and/or Canadian Cardiovascular Educational objective:
·1271 Society class Ill or IV angina while receiving medical therapy were Susceptibility bias is a type of selection bias where a treatment
·~ excluded from the study. (The Canadian Cardiovascular Society regimen is selected for a patient based on the severity of their
29
30 angina classification ranges from class 0. which indicates no condition. without taking into account other possible
31 symptoms, to class IV, which indicates angina at any level of confounding variables.
32 physical exertion.)
33 References:
34 Intervention: CABG plus optimal medical therapy vs optimal medical
therapy alone 1. The role of susceptibility bias in epidemiologic
35 research.
36 Outcome measures: The primary outcome was the rate of death
37
from any cause. Major secondary outcomes included the rates of
38
death from cardiovascular causes and of death from any cause or Time Spent 7 Copyright© Last updated:
39
hospitalization for cardiovascular causes. ~ seconds USMLEWorld ,LLC. [7/20/2013]
40
1.":~

2 Item: 5 of 44
Q.ld: 3885 (
11 PMark <:J
Previous
1>-
Ne><t
jf
Lab Values
9-•
Notes Calculator
I

3
4 A prospective cohort study revealed a strong positive association between smoking and liver cirrhosis (relative

6
=
risk 2 .8). The researchers then divided the cohort into two groups: alcohol consumers and
non-consumers. Subsequent statistical analysis did not reveal any association between smoking and liver
7 cirrhosis with either group. The scenario described above is an example of which of the following?
8
r A. Selection bias [12%]
·[;J
• 10 r B. Observer's bias [4%]
11 r C. Measurement bias [6%]
12
13 r D. Recall bias [2%]
14 .., r E. Confounding [76%]
15
16
17 Explanation: User ld:

·[m
• 19
Confounding refers to the bias that results when the exposure-disease relationship is mixed with the effect of
. 20
extraneous factors (i.e .. confounders). Confounders influence both the exposure and outcome. In the given
study, crude analysis of the data initially revealed an association between smoking and liver cirrhosis. The
•-~
most likely potential confounder is concomitant alcohol consumption in people who smoke. It is a well-known
22
fact that alcohol consumption is strongly associated with liver cirrhosis. Furthermore. alcohol consumption
23
can explain at least part of the association observed between smoking and liver cirrhosis.
24
25 Methods to deal with confounding include matching of cases and controls based on the confounding factor, or
26 stratification of the study population based on the confounding factor. In this case, running separate analyses
·c:]
• 28
for alcohol consumers and non-consumers (this technique is called stratified analysis) can unmask
confounding and disclose the true unconfounded value of the RR.
29
30 (Choice A) Selection bias results from the manner in which people are selected for the study, or from the
31 selective losses from follow-up. The scenario does not mention any of these problems.
32
33 (Choices B and C) Observer's bias and measurement bias distort the measure of association by
34 misclassifying exposed/unexposed and/or diseased/non-diseased subjects. The scenario does not describe
35 this classification process.
36
37 (Choice D) Recall bias results from the inaccurate recall of past exposure by people in the study. It applies
38 mostly to case-control studies. not cohort studies.
39
40 Educational Ojective:
1.":~

2 Item: 5 of 44
Q.ld: 3885 (
11 PMark <:1
Previous
1>-
Ne><t
jf
Lab Values
9''
Notes Calculator
I

3
4
r B. Observer's bias [4%]
6 r C. Measurement bias [6%]
7 r D. Recall bias [2%]
8
.., r E. Confounding [76%]
·[;J
• 10
11 Explanation: User ld:
12
13 Confounding refers to the bias that results when the exposure-disease relationship is mixed with the effect of
14 extraneous factors (i.e .. confounders). Confounders influence both the exposure and outcome. In the given
15 study, crude analysis of the data initially revealed an association between smoking and liver cirrhosis. The
16 most likely potential confounder is concomitant alcohol consumption in people who smoke. It is a well-known
17 factthat alcohol consumption is strongly associated with liver cirrhosis. Furthermore. alcohol consumption
·[m
• 19
can explain at least part of the association observed between smoking and liver cirrhosis.

. 20 Methods to deal with confounding include matching of cases and controls based on the confounding factor. or

•-~
stratification of the study population based on the confounding factor. In this case. running separate analyses
22 for alcohol consumers and non-consumers (this technique is called stratified analysis) can unmask
23 confounding and disclose the true unconfounded value of the RR.
24
25 (Choice A) Selection bias results from the manner in which people are selected for the study, or from the
26 selective losses from follow-up. The scenario does not mention any of these problems.

·c:]
• 28 (Choices B and C) Observer's bias and measurement bias distort the measure of association by
misclassifying exposed/unexposed and/or diseased/non-diseased subjects. The scenario does not describe
29
30 this classification process.
31
(Choice D) Recall bias results from the inaccurate recall of past exposure by people in the study. It applies
32
mostly to case-control studies. not cohort studies.
33
34 Educational Ojective:
35 Know the concept of confounding. Distinguish between crude and adjusted measures of association.
36 Confounding refers to the bias that can result when the exposure-disease relationship is mixed with the effect
37 of extraneous factors (i.e .. confounders).
38
39 Copyright© USMLEWorld ,LLC. Last updated [7/7/20 10]
Time Spent 2 seconds
40
1.":~
• 1
• 2 Item: 6 of 44 11 PMark <:1 1>- jf 9'' I

:CJ Q.ld: 3960 ( Previous Ne><t

A prospective cohort study was conducted to assess the relationship between caffeine consumption and the
risk of colon cancer in middle-aged women. The study showed that caffeine consumption decreases the risk
Lab Values Notes Calculator

of colon cancer with a relative risk of 0.83 and p value of 0.04. The factthatthe study was conducted

8
specifically in middle-aged women raises most concerns regarding which of the following issues? r
:[lJ r
., r
A. Bias [1 0%]
B. Generalizability [84%]
11
12 r C. Internal validity [3%]
13
14
r D. Reliability [3%]
15 r E. Reproducibility [1 %]
16
17
Explanation: User ld:
·liBl
·~ The generalizability, or external validity, of a study is defined as the applicability of the obtained results beyond
• 20
the cohort that was studied. External validity answers the question. "How generalizable are the results of a
·1211 study to other populations?" In this study, the external validity seems to be quite limited due to the restriction
. LBJ of the study population to middle-aged women. In other words. because the cohort is restricted to
23
middle-aged women. the results of the study are applicable only to middle-aged women. This is an important
24
concept when applying the results of observational studies and clinical trials to everyday practice if the original
25 inclusion and exclusion criteria are not reviewed carefully. The other answer options are relevant for internal
26
validity answering the following question: "Are the results obtained in this specific cohort valid?"
·1271
·~ (Choice A) In this scenario. there is no description of how the study was designed. conducted. and analyzed.
29 so it is not possible to determine whether the internal validity was violated (ie. if bias is present).
30
31 (Choice C) Within this cohort (middle-aged women). the study could be valid. This property is called internal
32 validity, or validity as generally taught
33
34 (Choices D and E) Reliability or reproducibility is the measure of random error.
35
36 Educational objective:
37 Generalizability or external validity pertains to the applicability of study results to other populations (eg. the
38 results of a study in middle-aged women would not be expected to be applicable to elderly men).
39
40 Time Soent: 2 seconds Coovriaht © USMLEWorld .LLC. Last uodated f7/15/20 131
1.":~
• 1
• 2 Item: 7 of 44 11 PMark <:J 1>- jf 9-• I

:CJ 5
Q.ld: <W79 ( Previous Ne><t

A study was conducted to assess the association between hormone replacement therapy (HRT) in
Lab Values Notes Calculator

post-menopausal women and the level of serum C-reactive protein (CRP). The data from the study are
6
presented below:
8 CRP high CRP normal Total
HRT 32 41 73
:[lJ NoHRT
Total
28
60
49
90
77
150
11
12
13 Which of the following is the best statistical method to assess the association between HRT and elevated
14 CRP levels?
15
16 r A. Two-sample z-test [6%]
17
r B. Two-sample t-test [33%]
·liBl
·~ ., r C. Chi-square test [53%]
• 20 r D. ANOVA [5%]
·1211 r E. Meta-analysis [2%]
. LBJ
23
24
Explanation: User ld:
25
26 The chi-square test is used to compare the proportions of a categorized outcome. In this case, the outcome
·1271 (serum CRP level) is categorized as either "high" and "normal," and then presented with the exposure ("HRT"
·~ or "no HRT") in a 2 x 2 table. In one of the commonly used chi-square tests, the observed values in each of
29 the cells are compared to expected (under the hypothesis of no association) values. If the difference between
30 the observed and expected values is large. an association between the exposure and the outcome is
31 assumed to be present.
32
33 (Choices A and B) The two-sample z-test and two-sample t-test are used to compare two means. not
34 proportions.
35
36 (Choice D) Analysis of variance (ANOVA) is used to compare the means of three or more variables.
37
38 (Choice E) Meta-analysis is an epidemiologic method of pooling the data from several studies to do an
39 analysis having a relatively big statistical power.
40
1.":~
• 1
• 2 Item: 7 of 44 11 PMark <:J 1>- jf 9-• I

:CJ 5
Q.ld: <W79 (
nr<e1
NoHRT
.OL.
28
... ,
49
,.,
77
Previous Ne><t Lab Values Notes Calculator

6
Total 60 90 150

8 Which of the following is the best statistical method to assess the association between HRT and elevated
CRP levels?
:[lJ r
11 r A. Two-sample z-test [6%]
12
r B. Two-sample t-test [33%]
13
14 ., r C. Chi-square test [53%]
15 r D. ANOVA [5%]
16
r E. Meta-analysis [2%]
17
·liBl
·~ Explanation: User ld:
• 20
·1211 The chi-square test is used to compare the proportions of a categorized outcome. In this case, the outcome
. LBJ (serum CRP level) is categorized as either "high" and "normal," and then presented with the exposure ("HRT"
23 or "no HRT") in a 2 x 2 table. In one of the commonly used chi-square tests, the observed values in each of
24 the cells are compared to expected (under the hypothesis of no association) values. If the difference between
25 the observed and expected values is large. an association between the exposure and the outcome is
26 assumed to be present.
·1271
·~ (Choices A and B) The two-sample z-test and two-sample t-test are used to compare two means. not
29 proportions.
30
31
(Choice D) Analysis of variance (ANOVA) is used to compare the means of three or more variables.
32
(Choice E) Meta-analysis is an epidemiologic method of pooling the data from several studies to do an
33
analysis having a relatively big statistical power.
34
35 Educational Objective:
36 The chi-square test is used to compare proportions. A 2 x 2 table may be used to compare the observed
37 values with the expected values.
38
39
Time Spent 1 seconds Copyright© USMLEWorld ,LLC. Last updated [7/26/20 13]
40
1.~
• 1
• 2 Item: 8 of 44 11 PMark <J C>- jf 9-• I

:CJ 5
Q.ld: 3886 ( Previous Ne><t

A new test is devised to detect HIV. It has a sensitivity of 90% and specificity of 80% compared to the gold
standard. Consider that the test is used in two populations: a population in Africa having an HIV prevalence of
Lab Values Notes Calculator

6
0.20 (20%). and a population in Asia having an HIV prevalence of 0.01 ( 1%). Which is the most accurate
statement concerning the new test?

r A. Sensitivity of the test is higher in the African population [8%]


r B. Specificity of the test is higher in the Asian population [4%]
12
., r C. Positive predictive value of the test is higher in the African population [78%]
13
14 r D. Negative predictive value of the test is lower in the Asian population [6%]
15 r E. The test is not reliable in the African population [3%]
16
17
·liBl Explanation: User ld:
·~ The traditional interpretation of positive predictive value (PPV) is the proportion of people with positive test
• 20
results that actually have the disease. It is easier to understand the concept in terms of probability: if a patient
·1211
. LBJ has a positive test result, what is the probability that he/she has the disease? PPV depends on the
prevalence of the disease (very important concept to remember). The more common the disease (e.g .. 20%
23
24 prevalence of HIV in the African population). the greater the probability that a patient from that population and
25 with a positive test actually has the disease (i.e .. has a true positive result). In the Asian population. the
26 probability of a false-positive result for a patient with a positive test result is higher, because of the low
prevalence of HIV.
·1271
·~ (Choices A and B) The sensitivity and specificity of a test do not depend on the prevalence of the disease in
29
the population.
30
31
(Choice D) Like positive predictive value. negative predictive value depends on the prevalence of the disease
32 in the population. but has an inverse association with the prevalence. As the prevalence of the disease
33
increases. the negative predictive value decreases because the probability of a true negative result for a
34
patient who tested positive is high in a population with a low prevalence of the disease (i.e .. the Asian
35
population example).
36
37 (Choice E) The statement that the test is not reliable in the African population is not correct, because the test
38 has high sensitivity and specificity.
39
40 ~,. .. ,.~tiftn~l nhiat"tiua·
1.~
• 1
• 2 Item: 8 of 44 11 PMark <J C>- jf 9-• I

:CJ 5
Q.ld: 3886 (

r
r
Previous Ne><t

A. Sensitivity of the test is higher in the African population [8%]


B. Specificity of the test is higher in the Asian population [4%]
Lab Values Notes Calculator

6
., r C. Positive predictive value of the test is higher in the African population [78%]
r D. Negative predictive value of the test is lower in the Asian population [6%]
r E. The test is not reliable in the African population [3%]

12 Explanation: User ld:


13
14 The traditional interpretation of positive predictive value (PPV) is the proportion of people with positive test
15 results that actually have the disease. It is easier to understand the concept in terms of probability: if a patient
16 has a positive test result, what is the probability that he/she has the disease? PPV depends on the
17 prevalence of the disease (very important concept to remember). The more common the disease (e.g .. 20%
·liBl prevalence of HIV in the African population). the greater the probability that a patient from that population and
·~ with a positive test actually has the disease (i.e .. has a true positive result). In the Asian population. the
• 20 probability of a false-positive result for a patient with a positive test result is higher, because of the low
·1211 prevalence of HIV.
. LBJ
23 (Choices A and B) The sensitivity and specificity of a test do not depend on the prevalence of the disease in
24 the population.
25
26
(Choice D) Like positive predictive value. negative predictive value depends on the prevalence of the disease
in the population. but has an inverse association with the prevalence. As the prevalence of the disease
·1271 increases. the negative predictive value decreases because the probability of a true negative result for a
·~ patient who tested positive is high in a population with a low prevalence of the disease (i.e .. the Asian
29
30 population example).
31
(Choice E) The statement that the test is not reliable in the African population is not correct, because the test
32
has high sensitivity and specificity.
33
34 Educational Objective:
35 Know the concept of positive predictive value (PPV). PPV depends on the prevalence of the disease of
36 interest in the population to which the test is applied. PPV increases with an increase in prevalence of
37 disease in the study population. For NPV, the inverse is true.
38
39
Time Spent 2 seconds Copyright© USMLEWorld ,LLC. Last updated [7/7/20 10]
40
• 1
• 2

:CJ 5
6
The drug ad is focused on Efrenzia, a novel anti-platelet agent for the treatment of acute coronary syndromes.
7
8 Item 1 of2

A 58-year-old man with a history of hypertension and type 2 diabetes mellitus comes to the emergency
11 department because of chest pain and diaphoresis. The symptoms started two hours ago and have a
12 stuttering course. He has never had similar symptoms before. In the emergency department, his
13 electrocardiogram shows horizontal ST segment depression in leads V1 to V4. He is given the appropriate
14 medical therapy including low-dose aspirin, and referred to the catheterization laboratory due to persistence of
15 his angina. Based on the information provided in the drug ad, giving the patient Efrenzia as opposed to
16 clopidogrel would most likely decrease the risk of developing which of the following subsequent events?
17
·liBl View Drug Ad
·~
• 20
·1211 r A. Cardiovascular death [21%]
. LBJ r B. Major bleeding [6%]
23 r C. Non-fatal stroke [14%]
24
25 ., r D. Recurrent myocardial infarction [59%]
26
·1271 Explanation: User ld:
·~
29 This drug ad is comparing the effect of administering Efrenzia vs. clopidogrel in combination with aspirin for
30
patients with acute coronary syndrome undergoing percutaneous coronary intervention, including those with
31
unstable angina (UA)/non-ST elevation myocardial infarction (NSTEMI) and ST-elevation Ml (STEMI). The
32 results are reported as the percentage of patients developing a composite endpoint of cardiovascular death,
33
non-fatal Ml, or nonfatal stroke over the subsequent 18 months. A subgroup analysis was also performed in
34
diabetics presenting with NSTEM/UA or STEML
35
36 There was a 2. 7% overall decrease of events in patients with STEMI ( 12.4% reduced to 9. 7%) and a 1.8%
37 decrease of events in UAINSTEMI patients ( 10. 7% reduced to 8.9%). The text under the section titled "Benefit
38 in STEMI and UAINSTEMI patients" indicates that the reason for the difference between the treatments was
39 primary due to a significant reduction in (recurrent) non-fatal Mls.
40
1.":~
• 1
• 2 Item: 9 of 44 11 PMark -<:J C>- jf ""• I

:CJ 5
Q.ld: 7708 ( Previous Ne><t
clopldogrel would most likely decrease the nsk ot developing which ot the tollow1ng subsequent events't
Lab Values Notes Calculator

6 View Drug Ad
7
8
r A. Cardiovascular death [21%]
r B. Major bleeding [6%]
11 r C. Non-fatal stroke [14%]
12
13
., r D. Recurrent myocardial infarction [59%]
14
15 Explanation: User ld:
r
16
17 This drug ad is comparing the effect of administering Efrenzia vs. clopidogrel in combination with aspirin for
·liBl patients with acute coronary syndrome undergoing percutaneous coronary intervention. including those with
·~ unstable angina (UA)/non-ST elevation myocardial infarction (NSTEMI) and ST-elevation Ml (STEMI). The
• 20 results are reported as the percentage of patients developing a composite endpoint of cardiovascular death.
·1211 non-fatal MI. or nonfatal stroke over the subsequent 18 months. A subgroup analysis was also performed in
. LBJ diabetics presenting with NSTEM/UA or STEML
23
24 There was a 2. 7% overall decrease of events in patients with STEMI ( 12.4% reduced to 9. 7%) and a 1.8%
25 decrease of events in UAINSTEMI patients ( 10. 7% reduced to 8.9%). The text under the section titled "Benefit
26 in STEMI and UAINSTEMI patients" indicates that the reason for the difference between the treatments was
·1271 primary due to a significant reduction in (recurrent) non-fatal Mls.
·~ (Choices A and C) The drug ad states under the section titled "Benefit in STEMI and UAINSTEMI patients"
29
30 that there was no significant difference between the treatments in cardiovascular death or non-fatal stroke in
31 both UAINSTEMI and STEMI patients.
32
33 (Choice B) Major bleeding was not part of the primary composite endpoint. Also. the safety data at the
34 bottom of the drug ad indicates that overall bleeding rates were significantly increased with Efrenzia.
35
Educational objective:
36
When comparing the effects of a treatment on patient outcomes. it is important to note all causes of the
37
differences.
38
39
40 Time Spent 83 seconds Copyright© USMLEWorld ,LLC. Last updated [5/2/20 13]
~· 1.":~
• 1
• 2 Item: 10 of 44 Il l' Mark <:1 C>- if I

:CJ 5
Q.ld: 7709 [
ltem2 of2
Previous Ne><t Lab Values Notes Calculator

6 Based on the drug ad data. how many diabetic patients undergoing percutaneous coronary intervention for
7 unstable angina (UA) or non-ST segment elevation myocardial infarction (NSTEMI) need to be treated with
Efrenzia to prevent one composite event as compared to clopidogrel?

View Drug Ad

12
r A. 5 [16%]
13
14 r B. 10[11%]
15 ., r C. 25 [59%]
16
r D. 50 [1 0%]
17
·liBl r E. 1DO [4%]
·~
• 20
Explanation: User ld:
·1211
. LBJ The number needed to treat (NNT) is defined as the number of people that need to receive a treatment to
23
prevent one defined event. It is calculated as the inverse of the absolute risk reduction (ARR). In this study.
24
diabetic patients with UAINSTEMI had a 16.3% incidence of composite events with clopidogrel + aspirin and
25 12.3% with Efrenzia + aspirin.
26
·1271 ARR =0.163-0.123 =0.04
·~
29 NNT =1/ARR =1/0.04 =25
30
31 As compared to clopidogrel. Efrenzia would need to be given to at least 25 diabetic patients with UAINSTEMI
32 who are undergoing percutaneous coronary intervention in order to prevent one composite event (Choices A,
33 8, D, and E).
34
35 Educational objective:
36 The NNT is defined as the number of people that need to receive a treatment to prevent one adverse event. It
37 is calculated as the inverse of the absolute risk reduction (ARR).
38
39 I References:
40
~
• 1
• 2 Item: 10 of 44 !II PMark <:1 (>- jf 9-• I

:CJ 5
Q.ld: 7709 [ Previous Ne><t
L...l I '-'1 IL.IU I.U J-'1 '-' Y '-'1 II. Ul 1'-' '-'UI I IJ-'U~II.'-' '-' Y '-'1 II. U~ '-'UI I IJ-'UI '-''-" I.U '-"UJ-'IUU\:::11 '-'1!
Lab Values Notes Calculator

View Drug Ad
6
7
r A. 5 [16%]
r B. 10[11%]
., r C. 25 [59%] r
12 r D. 50 [1 0%]
13
14
r E. 1DO [4%]
15
16
Explanation: User ld:
17
·liBl The number needed to treat (NNT) is defined as the number of people that need to receive a treatment to
·~ prevent one defined event. It is calculated as the inverse of the absolute risk reduction (ARR). In this study.
• 20 diabetic patients with UAINSTEMI had a 16.3% incidence of composite events with clopidogrel + aspirin and
·1211 12.3% with Efrenzia + aspirin .
. LBJ
23 ARR =0.163-0.123 =0.04
24
25 NNT =1/ARR =1/0.04 =25
26
As compared to clopidogrel. Efrenzia would need to be given to at least 25 diabetic patients with UAINSTEMI
·1271 who are undergoing percutaneous coronary intervention in order to prevent one composite event (Choices A,
·~ 8, D, and E).
29
30
Educational objective:
31
The NNT is defined as the number of people that need to receive a treatment to prevent one adverse event. It
32
is calculated as the inverse of the absolute risk reduction (ARR).
33
34
35 References:
36 1. How effective is that treatment? The number needed to treat.
37
38
39
Time Spent 20 seconds Copyright© USMLEWorld .LLC. Last updated [5/2/20 13]
40
.
1.":~

.
1
Item: 11 of 44 11 f>Mark <:::1 C>- jf 9-•
·w
2 I

Q.ld: 2135 [ Previous Ne><t Lab Values Notes Calculator


• 4
5
6 In clinical practice. the "CAGE" questionnaire is used to screen patients for alcoholism. When a patient
7 replies with 2 out of 4 positive responses to the "CAGE" questions. the test is considered to be positive for
8 alcoholism. If this criteria is changed so that 3 out of 4 positive responses to the "CAGE" questions label the
patient as alcoholic. what is the effect on the sensitivity and specificity of this test?

12
r A. Both sensitivity and specificity of the test will decrease [3%]
13 r B. Both sensitivity and specificity of the test will increase [7%]
14
., r C. Sensitivity will decrease but specificity will increase [73%]
15
16 r D. Sensitivity will increase but specificity will decrease [14%]
17 r E. Both sensitivity and specificity will remain the same [3%]
·liBl
·~
• 20 Explanation: User ld:
·1211
. LBJ Raising the cut-off point of a test will increase its specificity and decrease its sensitivity. In this particular
23 vignette. increasing the number of questions for a patientto be labelled as an alcoholic will result in fewer
24 people being identified as such. Consequently. false and true positives will decrease. while false and true
25 negatives will increase.
26
Educational Objective:
·1271 Raising the cut-off point (e.g .. increasing the inclusion criteria) of a screening test results in an increase in
·~ specificity and decrease in sensitivity.
29
30
31 Time Spent 2 seconds Copyright© USMLEWorld .LLC. Last updated [11/1 0/2011]
32
33
34
35
36
37
38
39
40
2 Item: 12 of 44 11 f> Mark -<J t:>- jf ~· ~~ I

:CJ
5
Q.ld: 3947 [ Previous Ne><t

A prospective cohort study was conducted to assess the effects of oral contraceptive use on the incidence of
breast cancer. Crude analysis of the study results suggests an association between the use of oral
Lab Values Notes Calculator

6 = =
contraceptives and breast cancer (relative risk [RR] 1.4. p 0.04). Further analysis shows that in women
7 =
with a family history of breast cancer. oral contraceptives increased the risk of breast cancer (RR 2.1 D. p =
8 = =
0.01 ). In women without a family history of breast cancer. no effect was observed (RR 1.05. p 0.80). The

:[lJ results of this study are best explained by which of the following?

11
r A. Confounding [53%]
., r B. Effect modification [38%]
14 r C. Latency period [3%]
15
r D. Observer bias [2%]
16
17 r E. Selection bias [4%]
·liBl
·~ Explanation: User ld:
• 20
·1211
. LBJ Confounding versus effect modification
23
24 Confounding Effect modification
25
26 Crude analysis Crude analysis
·1271
·~ Alcohol Significant association Bladder Use of oral
Significant association Breast
29 use ca ncer contraceptive cancer
30
31
32
33 Stratified by smoking Stratified by family history
34
Smokers: Family history of breast cancer:
35
36
37
38
Alcohol
use
No association
------------ ~
Bladder
cancer
Use of oral
contraceptive 1-------•
Strong association Breast
cancer
39
40
2 Item: 12 of 44 !il f>Mark -<J t:>- jf ~· ~]il I

:CJ
5
Q.ld: 3947 [ Previous

Confound ing versus effect modification


Ne><t Lab Values Notes Calculator

6 Confounding Effect modification


7
8 Crude analysis Crude analysis

:[lJ Alcohol Significant association Bladder Use of oral


Significant association
Breast
11
use ca ncer contraceptive cancer

14
15 Stratified by smoking Stratified by family history
r
16
17 Smokers: Family history of breast cancer:
·liBl No as.sociation Strong association
-~ Alcohol
------------ ~
Bladder Use of oral Breast
• 20 use cancer contraceptive cancer
·1211
. LBJ Non-smokers: No family history of breast cancer:
23
24 No association Bladder No asse<iatlon
Alcohol Use of oral Breast
25 use ------------· ca ncer contraceptive ------------· cancer
26
·1271 €>VSMLEWOI"W.ll{
-~
29
Effect modification results when an external variable positively or negatively impacts the effect of a risk factor
30
on the disease of interest. It can sometimes be confused with confounding. the bias that results when the
31
exposure-disease relationship is obscured by the effect of an extraneous factor that is associated with both
32
the exposure and disease. Effect modification can be distinguished from confounding by performing a
33
stratified analysis centering on the variable of interest. If the variable is a confounder. there will be no
34
significant difference in risk between the stratified groups as the confounding effects are now removed.
35
However. if the variable is instead an effect modifier. there will be a significant difference between the 2 groups.
36
37 In this case. stratification by family history shows that oral contraceptives significantly increase the risk of
38 breast cancer in patients with a positive family history but not in patients with a negative family history. Thus.
39 family history is not a confounding variable (Choice A). Rather. positive family history acts as an effect
40 .. .
2 Item: 12 of 44 11 PMark -<:J 1>- jf ~ ~~ I

:CJ
5
Q.ld: 3947 [
Non-smokers:
Previous Ne><t
No family history of breast cancer:
Lab Values Notes Calculator

Alcohol No association Bladder No association Breast


Use of oral
6
7 use ------------· ca ncer contraceptive ------------· cancer
8

:[lJ €>VSMLEW¢11tHL<

11 Effect modification results when an external variable positively or negatively impacts the effect of a risk factor
on the disease of interest. It can sometimes be confused with confounding. the bias that results when the
exposure-disease relationship is obscured by the effect of an extraneous factor that is associated with both
14 the exposure and disease. Effect modification can be distinguished from confounding by performing a
15 stratified analysis centering on the variable of interest. If the variable is a confounder. there will be no
16 significant difference in risk between the stratified groups as the confounding effects are now removed.
17 However. if the variable is instead an effect modifier. there will be a significant difference between the 2 groups.
·liBl
-~ In this case. stratification by family history shows that oral contraceptives significantly increase the risk of
• 20 breast cancer in patients with a positive family history but not in patients with a negative family history. Thus.
·1211 family history is not a confounding variable (Choice A). Rather. positive family history acts as an effect
. LBJ modifier to increase the risk of breast cancer in patients taking oral contraceptives. Other well-known
23 examples of effect modification include the effect of estrogens on the risk of venous thrombosis (augmented
24 by smoking) and the risk of lung cancer in people exposed to asbestos (also enhanced by smoking).
25
26
(Choice C) The latency period is the time required for a given exposure to have a measurable effect on the
outcome. This study provided no information on how long oral contraceptives must be used in order to have
·1271 an effect on breast cancer risk in susceptible patients.
-~
29
(Choices D and E) Flaws in a study involving subject enrollment or data recording can result in selection
30
bias or observer bias. respectively. Effect modification is not a bias. but rather a natural phenomenon that is
31
importantto recognize.
32
33 Educational objective:
34 Effect modification results when an external variable positively or negatively impacts the effect of a risk factor
35 on the disease of interest. It can be distinguished from confounding by performing a stratified analysis
36 centered on the variable of interest. Effect modification is not a bias. but rather is a natural phenomenon that
37 is important to recognize.
38
39
Time Spent 1 seconds Copyright© USMLEWorld .LLC. Last updated [7/12/20 13]
40
2 Item: 13 of 44 ill PMark <j C> jf ~· ~ I

:CJ
5
Q.ld: 3897 [ Previous

A new multidrug chemotherapy regimen significantly prolongs the survival in patients with lung cancer. If this
Ne><t Lab Values Notes Calculator

6 new regimen is widely implemented. what changes in the prevalence and incidence of lung cancer would you
7 expect?
8

:[lJ r
A. Incidence will decrease. prevalence will decrease [1 %]

11
12
B. Incidence will increase. prevalence will not increase [2%]
14 r
15
16
C. Incidence will not change. prevalence will increase [91%]
17
·liBl
·~
• 20
D. Incidence will not change. prevalence will not change [3%]
·1211 r
. LBJ
23
24 E. Incidence will decrease. prevalence will increase [2%]
25 r
26
·1271
·~
29 Explanation: User ld:
30
31 It is important to understand the difference between prevalence and incidence. the two basic measures of
32 disease occurrence. Incidence is the measure of new cases. the rapidity with which they are diagnosed.
33 Prevalence is the measure of the fofa/ number of cases at a particular point in time. Incidence answers the
34 question: how many new cases of the disease developed in a population during a particular period of time?
35 Prevalence answers the question: how many cases of the disease exist in a population at a particular point in
36 time? The relationship between prevalence and incidence can be expressed as:
37
38 Prevalence =(Incidence) x (Time)
39
40 "T"I-- : .. _:.1- .. - - -£- .1:----- ;_ •• -<~> -1- -·--·-.I 1•. • -·-· • I.: ...I - £ .... __ ..... -·-<~> I.---··--.,,_- .1:----- I.-- -I.·-- .1. •
2 Item: 13 of 44 !il f> Mark <:J C>- jf ~· 1.":~ I

:CJ
5
Q.ld: 3897 [ Previous Ne><t

D. Incidence will not change. prevalence will not change [3%]


Lab Values Notes Calculator

6 r
7
8
E. Incidence will decrease. prevalence will increase [2%]
:[lJ r
11
12

14 Explanation: User ld:


15
16
17
It is important to understand the difference between prevalence and incidence. the two basic measures of
disease occurrence. Incidence is the measure of new cases. the rapidity with which they are diagnosed. r
Prevalence is the measure of the total number of cases at a particular point in time. Incidence answers the
·liBl question: how many new cases of the disease developed in a population during a particular period of time?
·~ Prevalence answers the question: how many cases of the disease exist in a population at a particular point in
• 20
time? The relationship between prevalence and incidence can be expressed as:
·1211
. LBJ Prevalence =(Incidence) x (Time)
23
24
25 The incidence of a disease is not changed by any kind of treatment. because the disease has already
26 occurred when treatment is started. On the other hand. the prevalence may be affected by treatment of the
·1271 disease. In this case. treatment of an acute and rapidly fatal disease (e.g .. lung cancer) prolonged the survival
·~ of patients; however. such treatment did not cure the disease. This will result in more people having the said
29 disease at one point in time; hence. the prevalence will increase.
30
31
(Choices A, B and E) The incidence of a disease is not affected by the treatment.
32
(Choice D) This statement is not correct because a prolonged time of survival will increase the prevalence.
33
34 Educational objective:
35 Know the difference between incidence and prevalence. Incidence is the measure of new cases. the rapidity
36 with which they are diagnosed. Prevalence is the measure of the total number of cases at a particular point in
37 time.
38
39
Time Spent 2 seconds Copyright© USMLEWorld .LLC. Last updated [7/7/20 10]
40
~· !:':~
• 1
• 2 Item: 14 of 44 !ll f> Mark <J C>- jf I

:CJ 5
Q.ld: 4157 [ Previous Ne><t

A cohort study was conducted to assess the relationship between high saturated fat consumption and the
occurrence of colorectal carcinoma among women. A group of women aged 40-65 was selected. The
Lab Values Notes Calculator

6
7
8
baseline saturated fat consumption was calculated using a food questionnaire. and the cohort was followed
for seven years for the development of colon cancer. The study showed that women with high baseline r
saturated fat consumption have four times the risk of colorectal cancer in a 7-year period. compared to
:[lJ women with low fat consumption (RR = 4.0. 95% Cl = 1.5 - 6.5). According to the study results. what percent
of colorectal carcinoma in women with high fat consumption could be attributed to their diet?
11

r A. 25% [34%]
r B. 50% [8%]
.., r c. 75% [42%]
16
r D. 90% [8%]
17
·liBl r E. 100% [8%]
·~
• 20
Explanation: User ld:
·1211
. LBJ Attributable risk percent (ARP) or etiologic fraction is an important measure of the impact of a risk factor being
23
studied. ARP represents the excess risk in a population that can be explained by exposure to a particular risk
24
factor. It is calculated by subtracting the risk in the unexposed population (baseline risk) from the risk in the
25
exposed population. and dividing the result by the risk in the exposed population:
26
·1271 ARP =(risk in exposed- risk in unexposed)/risk in exposed.
·~
29
30
An easier way to calculate the ARP is to derive it from the relative risk (RR):
31
32 ARP = (RR - 1)/RR.
33
34 In this case. ARP = (4.0- 1)/4.0 = 0.75 (75%). According to the study results. 75% of colorectal carcinoma in
35 the high consumption group was attributable to high saturated fat intake.
36
37 Educational Objective:
38 ARP represents the excess risk in the exposed population that can be attributed to the risk factor. It can be
39 easily derived from the relative risk using the following formula ARP = (RR - 1)/RR.
40
~· !:':~
• 1
• 2 Item: 14 of 44 !ll f> Mark <J C>- jf I

:CJ 5
Q.ld: 4157 [ Previous Ne><t
occurrence or colorectal carc1noma among women. A group or women age a 4U-tio was selectea. 1ne
baseline saturated fat consumption was calculated using a food questionnaire. and the cohort was followed
Lab Values Notes Calculator

6 for seven years for the development of colon cancer. The study showed that women with high baseline
7 saturated fat consumption have four times the risk of colorectal cancer in a 7-year period. compared to
8 women with low fat consumption (RR = 4.0. 95% Cl = 1.5 - 6.5). According to the study results. what percent
of colorectal carcinoma in women with high fat consumption could be attributed to their diet?
r
:[lJ
11 r A. 25% [34%]
r B. 50% [8%]
.., r c. 75% [42%]
r D. 90% [8%]
16
r E. 100% [8%]
17
·liBl
·~ Explanation: User ld:
• 20
·1211 Attributable risk percent (ARP) or etiologic fraction is an important measure of the impact of a risk factor being
. LBJ studied. ARP represents the excess risk in a population that can be explained by exposure to a particular risk
23 factor. It is calculated by subtracting the risk in the unexposed population (baseline risk) from the risk in the
24 exposed population. and dividing the result by the risk in the exposed population:
25
26 ARP =(risk in exposed- risk in unexposed)/risk in exposed.
·1271
·~ An easier way to calculate the ARP is to derive it from the relative risk (RR):
29
30
ARP = (RR - 1)/RR.
31
32
In this case. ARP = (4.0- 1)/4.0 = 0.75 (75%). According to the study results. 75% of colorectal carcinoma in
33
the high consumption group was attributable to high saturated fat intake.
34
35 Educational Objective:
36 ARP represents the excess risk in the exposed population that can be attributed to the risk factor. It can be
37 easily derived from the relative risk using the following formula ARP = (RR - 1)/RR.
38
39
Time Spent 1 seconds Copyright© USMLEWorld .LLC. Last updated [7/7/20 10]
40
2 Item: 15 of 44 11 PMark -<J C>- jf 9'' 1.~ I

Q.ld: ~86 [

:CJ
5
Previous Ne><t

Over the last 5 years. more people in town A seem to be suffering from leukemia than in town B. Which of the
following study designs is the best for calculating the different incidences of leukemia in these two towns?
Lab Values Notes Calculator

6
7
r A. Case-control [16%]
8
., r B. Cohort [46%]
:[lJ r C. Case series [5%]
11
r D. Clinical trial [1 %]
12
13 r E. Cross-sectional [30%]
14

16
Explanation: User ld:
17
Incidence is defined as the frequency of new cases of a disease arising in a population at risk over a specified
·liBl time period. Thus. in order to determine incidence. subjects withoutthe disease must be followed over a
·~ period oftime to discover how many subjects develop said disease. Among the choices listed. the best
• 20
option for determining incidence is a cohort study. A cohort study is a prospective observational study in
·1211 which groups are chosen based upon presence or absence of one or more risk factors. All subjects are then
. LBJ observed over time for development of the disease of interest. allowing estimation of incidence within the total
23 population and comparison of incidences between subgroups. In this case. residents of town A and town B
24
could be followed for a period of time. and the incidence of leukemia determined for each site. Comparing the
25 incidences between the two towns will allow determination of the relative risk for developing leukemia in town
26
A versus town B.
·1271
·~ (Choice A) In a case control study. subjects with the disease of interest are compared to an otherwise similar
29 group that is disease free. Information is then collected about exposure to risk factors. A case control study
30
is retrospective and meantto determine associations between risk factors and disease occurrence. An odds
31
ratio can be calculated in a case control study. but the incidence of a disease cannot.
32
33
(Choice C) A case series is a study involving only patients already diagnosed with a condition of interest. A
34
case series can be helpful in determining the natural history of uncommon conditions but provides no
35
information about disease incidence.
36
37
(Choice D) A clinical trial compares the therapeutic benefit of different interventions in patients already
38
diagnosed with a particular disease. Clinical trials cannot be used to determine disease incidence.
39
40
2 Item: 15 of 44 !il f>Mark <:J C>- jf ~· 1.":~ I

Q.ld: ~86 [

:CJ
5
Previous Ne><t Lab Values Notes Calculator

Explanation: User ld:


6
7 Incidence is defined as the frequency of new cases of a disease arising in a population at risk over a specified
8 time period. Thus. in order to determine incidence. subjects without the disease must be followed over a
:[lJ period of time to discover how many subjects develop said disease. Among the choices listed. the best
option for determining incidence is a cohort study. A cohort study is a prospective observational study in
11 which groups are chosen based upon presence or absence of one or more risk factors. All subjects are then
12
observed over time for development of the disease of interest. allowing estimation of incidence within the total
13 population and comparison of incidences between subgroups. In this case. residents of town A and town B
14
could be followed for a period of time. and the incidence of leukemia determined for each site. Comparing the
incidences between the two towns will allow determination of the relative risk for developing leukemia in town
16 A versus town B.
17
·liBl (Choice A) In a case control study. subjects with the disease of interest are compared to an otherwise similar
·~ group that is disease free. Information is then collected about exposure to risk factors. A case control study
• 20 is retrospective and meant to determine associations between risk factors and disease occurrence. An odds
·1211 ratio can be calculated in a case control study. but the incidence of a disease cannot.
. LBJ
23 (Choice C) A case series is a study involving only patients already diagnosed with a condition of interest. A
24 case series can be helpful in determining the natural history of uncommon conditions but provides no
25 information about disease incidence.
26
·1271 (Choice D) A clinical trial compares the therapeutic benefit of different interventions in patients already
·~ diagnosed with a particular disease. Clinical trials cannot be used to determine disease incidence.
29
30 (Choice E) A cross-sectional study takes a sample of individuals from a population at one point in time. It
31 allows determination of a disease·s prevalence (the total number of cases in a population at a given
32 time). Since subjects are not studied longitudinally. disease incidence cannot not be determined.
33
34 Educational objective:
35 A prospective cohort study design is best for determining the incidence of a disease. Comparing the
36 incidence of the disease in two populations (one with and one without a given risk factor) allows for calculation
37 of a relative risk.
38
39
Time Spent 1 seconds Copyright© USMLEWorld .LLC. Last updated [7/7/20 10]
40
2 Item: 16 of 44 11 PMark -<J C>- jf ""• 1.":~ I

:CJ
5
Q.ld: 3931 [ Previous Ne><t

A study was conducted to assess the association between L-tryptophan use and the development of
Lab Values Notes Calculator

6
Eosinophilia-Myalgia Syndrome (EMS). Patients with EMS were asked aboutthe use of products containing
7
L-tryptophan during the last 6 months. Atthe same time. people without EMS were randomly selected from
8
the same population where the patients came from. and asked abouttheir experience with L-tryptophan
containing products within the last 6 months. The study showed that the use of L-tryptophan is significantly
:[lJ associated with EMS. Which of the following measures of association are the investigators most likely to
report?
11
12
13 r A. Relative risk [27%]
r B. Median survival [1 %]
., r C. Exposure odds ratio [63%]
17 r D. Relative rate [3%]
·liBl r E. Prevalence odds ratio [5%]
·~
• 20
·1211 Explanation: User ld:
. LBJ
23 The above case describes a typical case-control study design. Patients with the disease of interest (cases)
24 and people without the disease (controls) are asked about previous exposure to the variable being studied
25 (L-tryptophan use). The main measure of association is the exposure odds ratio. in which the exposure of
26 people with the disease (cases) is compared to the exposure of those without the disease (controls).
·1271 (Choices A and D) Incidence measures (e.g .. relative risk or relative rate) cannot be directly measured in
·~ case-control studies because the people being studied are those who have already developed the disease.
29
30 Relative risk and relative rate are calculated in cohort studies. where people are followed over time for the
31 occurrence of the disease.
32
(Choice B) Median survival is calculated in cohort studies or clinical trials. and is usually used to compare the
33
34 median survival times in two or more groups of patients (e.g .. receiving a new treatment or placebo).
35
(Choice E) Prevalence odds ratio is calculated in cross-sectional studies to compare the prevalence of a
36 disease between different populations.
37
38 Educational Objective:
39 A case-control study is used to compare the exposure of people with the disease (cases) to the exposure of
40
2 Item: 16 of 44 11 PMark -<J C>- jf ""• 1.":~ I

:CJ
5
Q.ld: 3931 [ Previous Ne><t
L-rrypropnan aunng rne 1asr o monrns. AI rne same II me. people w1rnour t:IVI::> were ranaom1y se1ecrea rrom
the same population where the patients came from. and asked abouttheir experience with L-tryptophan
Lab Values Notes Calculator

6 containing products within the last 6 months. The study showed that the use of L-tryptophan is significantly
7 associated with EMS. Which of the following measures of association are the investigators most likely to
8 report?

:[lJ r A. Relative risk [27%]


11
r B. Median survival [1 %]
12
13 ., r C. Exposure odds ratio [63%]
r D. Relative rate [3%]
r E. Prevalence odds ratio [5%]

17
·liBl Explanation: User ld:
·~
• 20 The above case describes a typical case-control study design. Patients with the disease of interest (cases)
·1211 and people without the disease (controls) are asked about previous exposure to the variable being studied
. LBJ (L-tryptophan use). The main measure of association is the exposure odds ratio. in which the exposure of
23 people with the disease (cases) is compared to the exposure of those without the disease (controls).
24
25 (Choices A and D) Incidence measures (e.g .. relative risk or relative rate) cannot be directly measured in
26 case-control studies because the people being studied are those who have already developed the disease.
·1271 Relative risk and relative rate are calculated in cohort studies. where people are followed over time for the
·~ occurrence of the disease.
29
30 (Choice B) Median survival is calculated in cohort studies or clinical trials. and is usually used to compare the
31 median survival times in two or more groups of patients (e.g .. receiving a new treatment or placebo).
32
(Choice E) Prevalence odds ratio is calculated in cross-sectional studies to compare the prevalence of a
33
disease between different populations.
34
35 Educational Objective:
36 A case-control study is used to compare the exposure of people with the disease (cases) to the exposure of
37 the people without the disease (controls). The main measure of association is the exposure odds ratio.
38
39
Time Spent 2 seconds Copyright© USMLEWorld .LLC. Last updated [7/7/20 10]
40
1.~
• 1
• 2 Item: 17 of 44 11 PMark <J C>- jf 9-• I

:CJ 5
Q.ld : 3934 [ Previous Ne><t

A group of investigators conducted a randomized placebo-controlled clinical trial to assess the effect of a new
aldosterone-receptor antagonist on the progression of chronic heart failure. The primary outcome in this
Lab Values Notes Calculator

6
7
8
study was an all-cause mortality. They reported a decrease in all-cause mortality in the treatment group with
a relative risk of 0. 71 (p = 0.01). Which of the following statements is the best interpretation of the reported
association?
r
:[lJ r A. There is only a 1% chance thatthe result is biased [4%]
11
12 r B. The 95% confidence interval for the relative risk includes 1.0 [6%]
13 ., r C. There is 1% probability that the result was observed by chance [75%]
14
r D. There is a 71% decrease in all-cause mortality due to the drug [8%]
15
16 r E. The results obtained are not statistically significant [7%]

Explanation: User ld:


20
Know how to interpretthe 'p' value in conjunction with a measure of association (e.g .. relative risk). Relative
·1211 risk is a point estimate of association. but it does not account for random error. In other words. it is always
. LBJ possible that the calculated relative risk occured by chance. The 'p' value is used to strengthen the results of
23
the study; it is defined as the probability of obtaining the result by chance alone. In this scenario. the
24
probability that the result was obtained by chance is 1% (i.e .. p=O .01). The commonly accepted upper limit
25
(cut-off point) of the 'p' value for the study to be considered statistically significant is 0.05 (i.e .. less then 5%).
26
·1271 (Choice A) The 'p' value deals with random variability. not bias. which is systematic error.
·~
29 (Choice B) If the 'p' value is less than 0.05. the 95% confidence interval does not contain 1.0 (a 'null' value for
30 relative risk).
31
32 (Choice D) A relative risk of 0. 71 shows thatthe drug decreased the risk of mortality by 29% (again. a null
33 value for relative risk is 1.0).
34
35 Educational Objective:
36 The p value is the probability that the result of a study was obtained by chance alone. A study is considered
37 statistically significant when the 'p' value used is less than 0.05. Know how to interpretthe 'p' value and its
38 relationship with the confidence interval.
39
40 Time Soent: 2 seconds Coovriaht © USMLEWorld .LLC. Last uodated f7/7/20 101
2 Item: 18 of 44 !II PMark <J C>- jf ~· 1.":~ I

:CJ
5
Q.ld: 7686 [ Previous

The drug ad in the exhibit button applies to the next 2 items


Ne><t Lab Values Notes Calculator

6
7 The drug ad is focused on kalgatran (Kalaxin). a new oral anticoagulant for the treatment of non-valvular atrial
8 fibrillation.

:[lJ Item 1 of2


11
Bleeding risk is the major concern when anticoagulating patients with non-valvular atrial fibrillation. The ad
12
promotes Kalaxin as a safe medication. In the study comparing Kalaxin and warfarin. which of the following
13
specific bleeding risks were most similar between the two groups?
14
15
View Drug Ad

r A. Intracranial bleeding [1 0%]


r B. Gastrointestinal bleeding [8%]
r C. Life-threatening bleeding [23%]
., r D. Major bleeding [37%]
r E. Total bleeding [21%]
24
25
26 Explanation: User ld:
·1271
·~ This drug ad is comparing the effect of Kalaxin versus warfarin for preventing stroke in non-valvular atrial
29 fibrillation. It compares the hazard rates for adverse events for both drugs in the form of hazard ratios. A
30 hazard rate is the chance of an event occurring in either the treatment arm or control arm over a set period of
31 time. A hazard ratio is the chance of an event occurring in the treatment arm compared to the chance of that
32 event occurring in the control arm during a set period of time.
33
34 A hazard ratio < 1 indicates that an event is more likely to occur in the control arm. A hazard ratio > 1 signifies
35 that an event is more likely to occur in the treatment arm. A ratio close to 1 implies little difference between
36 the two groups. In this study. the hazard ratio for major bleeding was 0.96 (given in the bleeding statistics
37 under the chart). which is the closest to 1 compared to the other answer options. Additionally. the 95%
38 confidence interval (0 .84 - 1.1 D) contains the null value of 1. indicating that there is no significant difference in
39 the risk of major bleeding between the 2 groups.
40
1.":~
• 1
• 2 Item: 18 of 44 11 PMark <:J 1>- jf 9-• I

:CJ 5
Q.ld: 7686 [

r C. Life-threatening bleeding [23%]


.., r D. Major bleeding [37%]
Previous Ne><t Lab Values Notes Calculator

6
7 r E. Total bleeding [21%]
8

:[lJ Explanation: User ld:


11 This drug ad is comparing the effect of Kalaxin versus warfarin for preventing stroke in non-valvular atrial
12 fibrillation. It compares the hazard rates for adverse events for both drugs in the form of hazard ratios. A
13 hazard rate is the chance of an event occurring in either the treatment arm or control arm over a set period of
14 time. A hazard ratio is the chance of an event occurring in the treatment arm compared to the chance of that
15 event occurring in the control arm during a set period of time.

A hazard ratio < 1 indicates that an event is more likely to occur in the control arm. A hazard ratio > 1 signifies
that an event is more likely to occur in the treatment arm. A ratio close to 1 implies little difference between
the two groups. In this study. the hazard ratio for major bleeding was 0.96 (given in the bleeding statistics
under the chart). which is the closestto 1 compared to the other answer options. Additionally. the 95%
confidence interval (0 .84 - 1.1 D) contains the null value of 1. indicating thatthere is no significant difference in
the risk of major bleeding between the 2 groups.

24 (Choice A) The hazard ratio for intracranial bleeding is 0.39. indicating that Kalaxin has a lower chance of
25 causing intracranial bleeding than warfarin.
26
·1271 (Choice B) The hazard ratio for gastrointestinal (GI) bleeding is 1 75. indicating that Kalaxin has a higher
·~ chance of causing gastrointestinal bleeding than warfarin. The hazard ratio for major Gl bleeding is 1.38.
29
30 (Choice C) The hazard ratio for life-threatening bleeding is 0.75. indicating that Kalaxin has a lower chance of
31
causing life-threatening bleeding than warfarin.
32
(Choice E) The hazard ratio for total bleeding is 0.91 . indicating that Kalaxin has a slightly lower chance of
33
causing overall bleeding than warfarin.
34
35 Educational objective:
36 Hazard ratios are proportions that indicate the chance of an event occurring in the treatment arm as
37 compared to the chance of the event occurring in the control arm.
38
39
Time Spent 52 seconds Copyright© USMLEWorld .LLC. Last updated [5/2/20 13]
40
2 Item: 19 of 44 11 PMark -<J C>- jf ""• 1.":~ I

:CJ
5
Q.ld: 7687 [

ltem2 of2
Previous Ne><t Lab Values Notes Calculator

6 Proper randomization when assigning patients to treatment and control groups is important in order to avoid
7 the effect of extraneous variables on the study results. In the current study. which of the following forms of
8 additional information would help most in determining randomization success?

:[lJ View Drug Ad


11
12
13
r A. Annual stroke rates [2%]
14 ., r B. Baseline characteristics [67%]
15 r C. Patient compliance chart [7%]
16
17
r D. Patient follow-up rates [7%]
r E. Subgroup analysis tables [17%]

• 20
Explanation: User ld:
·1211
. LBJ In any randomized clinical study. the goal of successful randomization is to eliminate bias in treatment
23
assignments in order to minimize confounding variables. An ideal randomization process minimizes selection
24
bias. results in near equal patient group sizes. and achieves a low probability of confounding variables. This
25
allows a study to achieve adequate statistical power. A listing of the baseline characteristics of the patients in
26
each arm would demonstrate if the two arms had patients with similar characteristics and would ensure that
·1271 proper randomization occurred in the study.
·~
29 (Choice A) Annual stroke rate is an endpoint of the study that may be influenced by patient randomization.
30 but it does not determine success of randomization.
31
32 (Choice C) Patient compliance charts indicate how well a patient tolerated the treatment and followed the
33 treatment regimen. but they would not directly indicate the success of randomization.
34
35 (Choice D) Patient follow-up rates indicate how many patients followed-up or dropped out of the study. This
36 data is not a direct marker of randomization success.
37
38 (Choice E) Subgroup analysis tables (stratified analysis) are used to determine if certain patient
39 characteristics (eg. age or comorbidities) influenced treatmentto any significant extent. Improper
40 randomization can result in inaccurate results durinq subqroup analvsis.
2 Item: 19 of 44 11 PMark <:1 (:>- if "!J'• 1.":~ I

:CJ
5
Q.ld: 7687 [

r D. Patient follow-up rates [7%]


Previous Ne><t Lab Values Notes Calculator

6 r E. Subgroup analysis tables [17%]


7
8
Explanation: User ld:
:[lJ In any randomized clinical study. the goal of successful randomization is to eliminate bias in treatment
11 assignments in order to minimize confounding variables. An ideal randomization process minimizes selection
12
bias. results in near equal patient group sizes. and achieves a low probability of confounding variables. This
13 allows a study to achieve adequate statistical power. A listing of the baseline characteristics of the patients in
14 each arm would demonstrate if the two arms had patients with similar characteristics and would ensure that
15 proper randomization occurred in the study.
16
17 (Choice A) Annual stroke rate is an endpoint of the study that may be influenced by patient randomization.
but it does not determine success of randomization.

• 20 (Choice C) Patient compliance charts indicate how well a patient tolerated the treatment and followed the
·1211 treatment regimen. but they would not directly indicate the success of randomization .
. LBJ
23 (Choice D) Patient follow-up rates indicate how many patients followed-up or dropped out of the study. This
24 data is not a direct marker of randomization success.
25
26 (Choice E) Subgroup analysis tables (stratified analysis) are used to determine if certain patient
characteristics (eg. age or comorbidities) influenced treatment to any significant extent. Improper
·1271 randomization can result in inaccurate results during subgroup analysis.
·~
29
Educational objective:
30
Successful randomization in a clinical trial allows a study to achieve adequate statistical power. An ideal
31
randomization process minimizes selection bias. results in near equal patient group sizes. and achieves a
32
low probability of confounding variables.
33
34
35 References:
36 1. Statistical properties of randomization in clinical trials.
37
38
39
Time Spent 2 seconds Copyright© USMLEWorld .LLC. Last updated [4/25120 13]
40
2 Item: 20 of 44 IJIII f>Mark <:1 f>- jf ~· l:'::.'ll I

:CJ
5
Q.ld: 2137 [ Previous Next

A gynecologic oncology research institute isolates a potential tumor marker for endometrial cancer. A large
multicenter study is then performed to evaluate serum levels of the tumor marker in women with and without
Lab Values Notes Calculator

6 endometrial cancer. The following curves are generated using the results of the study.
7
8

:[lJ 0 Healthy A B
11
12
D Diseased
13 ....<=
Ill

14 Ql

15
·.::
..."'...a.
0
Ql
..c
E
:::l
z

24
25
26 Ant igen concentration
·1271 ($) USMLEWorld, LLC

·~
29 Clinical researchers decide to use the tumor antigen to develop a confirmatory test for patients with
30 suspected endometrial cancer. During preliminary design of the test, the cutoff point for positive/negative
31 results is set at point A. If the cutoff point is moved from A to B, the specificity of the test will change in which
32 of the following ways?
33
34
35
r A. Cannot be determined [0%]
36 r B. Decreased [15%]
37 .., r C. Increased [83%]
38
39
r D. Unchanged [2%]
40
~· 1.":~
• 1
• 2 Item: 20 of 44 ill PMark <:J C>- jf I

:CJ 5
Q.ld: 2137 [

Test cutoff
Previous Ne><t Lab Values Notes Calculator

6
7
8 I - - Negative test ---il 1
l-- - Positive test - ---;
:[lJ ...."'c
11 41
·;;;
12
13
14
..."'...c.
0
41
15 ..c
E
::l
z

r
Ant igen concentration
© U SMLEWo~d.LLC
24
25
26 Important parameters of diagnostic tests include the following:
·1271 • True positive (TP)- A patient with a positive test result who actually has the disease.
·~ • False positive (FP) - A patient with a positive test result who actually does not have the disease.
29 • True negative (TN)- A patient with a negative test result who actually does not have the disease.
30 • False negative (FN) - A patient with a negative test result who actually has the disease.
31 • Sensitivity- The number of patients correctly testing positive for a disease divided by the total number
32 of patients with the disease (TP I [TP + FN]). High sensitivity means that negative results are less
33 likely to be FNs and more likely to be TNs; thus, a negative result is better able to rule out the disease
34 (SnOut). Because tests with high sensitivity will correctly identify most of the patients with the
35 disease, sensitivity is very important for screening tests (which need to minimize FNs).
36 • Specificity- The number of patients correctly testing negative for a disease divided by the total number
37 of patients without the disease (TN I [TN + FP]). High specificity means that positive results are less
38 likely to be FPs and more likely to be TPs; thus, a positive result is better able to rule in the disease
39 (Spin). Because tests with high specificity will correctly identify most of the patients without the
40 _,; _____ -·---:.t::_: .... ;_. ·-·-. :·--·--·..&.-.. •• - • • • r. ..... _ .. ____ .. _ _ .. _ / ...•. :_,_ ·----· ... _ ... : .. : ... : __ .. P\_\
<~>£-
~· 1.~
• 1
• 2 Item: 20 of 44 11 PMark -<:J C>- jf I

:CJ Q.ld: 2137 [ Previous Ne><t Lab Values Notes Calculator

5
6
7
l/ Antigen concentration "-·
8

:[lJ © U SMLEWo~d.LLC

11
12
Important parameters of diagnostic tests include the following:
13 • True positive (TP)- A patient with a positive test result who actually has the disease.
14 • False positive (FP) - A patient with a positive test result who actually does not have the disease.
15 • True negative (TN)- A patient with a negative test result who actually does not have the disease.
• False negative (FN) - A patient with a negative test result who actually has the disease.
• Sensitivity- The number of patients correctly testing positive for a disease divided by the total number
of patients with the disease (TP I [TP + FN]). High sensitivity means that negative results are less
likely to be FNs and more likely to be TNs; thus. a negative result is better able to rule out the disease
(SnOut). Because tests with high sensitivity will correctly identify most of the patients with the
disease. sensitivity is very important for screening tests (which need to minimize FNs).
• Specificity- The number of patients correctly testing negative for a disease divided by the total number
of patients without the disease (TN I [TN + FP]). High specificity means that positive results are less
24 likely to be FPs and more likely to be TPs; thus. a positive result is better able to rule in the disease
25 (Spin). Because tests with high specificity will correctly identify most of the patients without the
26 disease. specificity is very important for confirmatory tests (which need to minimize FPs).
·1271
·~ In this example. moving the cutoff point from A to B will cause more patients without the disease to test
29
negative (fewer FPs). increasing the specificity of the test. However. as a consequence. more patients with
30
the disease will also test negative (more FNs). resulting in decreased sensitivity. Conversely. moving the
31
cutoff point in the other direction (eg. B to A) will increase the number of FPs and decrease the number of
32
FNs. decreasing specificity while increasing sensitivity.
33
34 Educational objective:
35 Changing the cutoff point of a quantitative diagnostic test will inversely affect its sensitivity and specificity.
36 Typically. raising the cutoff value will increase specificity (fewer false positives) and decrease sensitivity (more
37 false negatives). Screening tests need high sensitivity; confirmatory tests need high specificity.
38
39
Time Spent 13 seconds Copyright© USMLEWorld .LLC. Last updated [7122120 13]
40
2 Item: 21 of 44 11 f>Mark -<:1 C>- G
::::ll rs
.~· r.~

:CJ
5
Q.ld: 7711 [ Previous Ne><t

A 60-year-old male with known coronary artery disease is seen


during routine check-up. He presents with no complaints. He
Lab Values Notes Calculator

6 Objective: denies any abdominal pain. rectal bleeding. or weight loss. He


7
has no family history of colon cancer. His current medications
8 To assess the association of low-dose aspirin use with the
include metoprolol. atorvastatin. and low-dose aspirin. Based
:[lJ performance of two quantitative immunochemical fecal occult blood
tests (iF OBTs) in a large sample of patients undergoing colorectal
on the study results which of the following is the best statement
concerning the use of iF OBT in this patient?
11 cancer screening
12
13 Methods: A. Accuracy of the test of test is not affected by use of
14 r aspirin [7%]
Design: Cross-sectional study
15 r B. Use of low-dose aspirin decreases the sensitivity of
16 the test [20%]
Blinding Technician performing iF OBT tests was blinded to
17
colonoscopy results ., r C. Use of low-dose aspirin increases the sensitivity of
·liBl the test [69%]
·~ Setting: 20 gastroenterology practices in southern Germany
• 20 r D. Use of low-dose aspirin increases the specificity of
Patients: Patients undergoing screening colonoscopy (mean age the test [5%]
62.1 years) were asked to provide stool samples before the
23 procedure. Information about medication use was collected using a
24 Explanation: User ld:
standardized questionnaire. Specific exclusion criteria included the
25 following: visible rectal bleeding or previous positive F OBT result. The study above is looking at the effect of aspirin on the ability
26 history of inflammatory bowel disease. colonoscopy in the past 5 of two different assays to detect colorectal cancer. with
·1271 years. incomplete colonoscopy. inadequate bowel preparation for colonoscopy used as the gold standard to detect the
·~ colonoscopy. participants with pseudopolyps or histologically lesions. Sensitivity is a measure of the true positive rate and
29 undefined polyps at screening colonoscopy. and participants who indicates how well a test can detect those patients with a
30 reported regular use of high-dose analgesics. disease. A higher sensitivity makes it less likely thatthere are
31
false negatives. and more likely that a negative test result
32 Intervention: Two automated. enzyme-linked immunosorbent assay
indicates that a patient does not have a given
33 (ELISA) based iF OBTs (hemoglobin test and
condition. Specificity is a measure of the true negative rate and
34 hemoglobin-haptoglobin test) were performed before screening
indicates how well a test can rule out a given condition. The
35 colonoscopy.
higher the specificity. the less likely there are false positives
36 and the more likely that a positive test result confirms the
37 Outcome measures: Sensitivity. specificity. positive and negative
predictive values. and area under receiver operating characteristic presence of a condition.
38
39 (ROC) curves in detecting advanced colorectal neoplasms with two
A Receiver Operating Characteristic (ROC) curve plots the true
40 quantitative iF OBTs usinq colonoscopv as qold standard. ...:.] nnc-iti\ 10 r-:.to ( c-onc-iti\ ,jt-, '' -:.n-:.inc-t tho f-:.lc-o nnc-iti\ •o r-:.to ( 1
2 Item: 21 of 44
Q.ld : 7711 [
ill P M ark <:l
.
t>- I ..

!:"~ I

:CJ
5
Results.
I Prevtous Next

p g ( ) p
lab Values

positive rate (sensitivity) againstthe false positive rate ( 1 -


Notes Calculator

6 specificity. or inverse true negative rate). at different cutoff


1979 patients (233 regular users of low-dose aspirin and 1746 who
7 points for a given diagnostic test. This curve usually shows
never used low-dose aspirin) were studied. Advanced neoplasms
8 that an increase in sensitivity is offset by a decrease in
were found in 24 users ( 10.3%) and 181 nonusers ( 10 A%) of
:[lJ low-dose aspirin. specificity.

11 The figure above shows the ideal diagnostic test ( 1DO %


12 sensitive and specific). which provides the most useful
Figure 3 . Receiver Operating Characteristic Curves
13 for Detecting Advanced Colorectal Neoplasms by information. A diagnostic test that provides no useful
14 Quanti tative lmmunochemical Fecal Occult B lood information usually produces the diagnosis by random chance
15 Test According to Use of Low-Dose Aspi rin and demonstrates an inverse linear relationship between
16 sensitivity and specificity.
17
·liBl
All participants
r
·~ Hemoglobin test
• 20
Specificity, %
100 80 60 40 20 0
23
24
25
26
80
·c:]
• 28 .....................
29
30 #. 60 ..···.···
31 ~
32 .~

33
=(/)
cQ)
////
34 (/)
40
35
36
37
100 80 60 40 20
20 ...............
38 Specificity (%)
39 ...·······'
40 ~ Accuracv is defined as the proportion of true results (true


1
2 Item: 21 of 44 • I· Ma•k <:::1 [::> jf ~· l~:'ll

:CJ O.ld: 7711 ( Previous NeHt Lab Values Notes Calculato1·

5
6
F1gure 3 . Rece1ver Operating Chara ctenst1c Cu rves
for Detec ting Advance d Colore ctal Neoplasms by
Quantitative lmmun ochemical F eca l Occ ult Blood
0 ~./ I I I
7 Test Acco rding to Use of Low -Dos e Aspirin 100 80 60 40 20
8
Specificity (%)
:[lJ All partici pants

Accuracy is defined as the proportion of true results (true


11 Hemoglobin test
12 positive and true negative) out of all the results that are
13
SpecifiCity, % predicted by a test in a given population. as measured by a gold
14 100 80 60 40 20 0 standard or reference parameter. The closer the plotted curve
15 approaches the left and top borders of the ROC curve. the
16 more accurate the test is. Accuracy can also be measured as
17 the total area under the plotted curve. Precision is defined as

:I::I
• 20
80
2 1 ,.----~
the proportion of true positives out of the total number of
positive results produced by a test in a given
population. Precision is equivalent to positive predictive
value . Both accuracy and precision depend upon the sensitivity
... .·· and specificity of the test. as well as the prevalence of the
....·· condition in the population being tested .
..··•·
..•· In this study , aspirin use moves the ROC curve upwards. This
..·..··
..·· translates to an increase in sensitivity(Choice 8). However,
26
the curve also shifts to the right for a given cutoff point with
·1271 ...........···••····•
aspirin use . indicating a decrease in the specificity (Choice
-~ D). The total area under the plotted curve increases with
29 20
30
.................... aspirin use (despite the decrease in specificity, as the
magnitude increase in sensitivity is larger). leading to an
31 .······
..···· increase in overall accuracy (Choice A) .
32
33
Educational objective:
34 0 20 40 60 80 100
A shift in the ROC curve upwards for a given cutoff indicates
35 100-Specificity, % increased sensitivity. A shift of the curve to the right for a given
36
cutoff point indicates a decrease in specificity.
37
38 Hemoglobin-haptoglobin test
Time Spent 40 Copyright® Last updated:
39 Specificity, % .:::J seconds USMLEWor1d,LLC. [31712013)
40
~ ~
• 1
• 2 Item: 21 of 44 !il PMark <:1 t> if I

:CJ Q.ld: 7711 [ Previous Next Lab Values Notes Calculator

5
6 Hemoglobin-haptoglobin test 0 ~/ I I I I
7 Specificity, % 100 80 60 40 20
8
100 80 60 40 20 0 Specificity (%)
:[lJ Accuracy is defined as the proportion of true results (true
11
12 positive and true negative) out of all the results that are
13 predicted by a test in a given population, as measured by a gold
80 ...········ standard or reference parameter. The closer the plotted curve
14
15 ..···· approaches the left and top borders of the ROC curve, the
16 more accurate the test is. Accuracy can also be measured as
................... the total area under the plotted curve. Precision is defined as
17
the proportion of true positives out of the total number of
·liBl ..·•···
positive results produced by a test in a given
·~ ..·•·····•
population. Precision is equivalentto positive predictive
• 20
..········· value. Both accuracy and precision depend upon the sensitivity
....····· and specificity of the test, as well as the prevalence of the
..•··
23 condition in the population being tested.
...... , . . . - - - - - - - - - - - - ,
24 20 Cut points, 1-fg/9 stool
25 ......................... In this study, aspirin use moves the ROC curve upwards. This
• Users of low-dose aspirin translates to an increase in sensitivity(Choice B). However,
26
..•·· o Nonusers of low-dose aspirin the curve also shifts to the right for a given cutoff point with
·1271 ..•·· aspirin use, indicating a decrease in the specificity (Choice
·~ D). The total area under the plotted curve increases with
29 0 20 40 60 80 100
aspirin use (despite the decrease in specificity, as the
30 1oo - Specificity, %
31
magnitude increase in sensitivity is larger), leading to an
32 increase in overall accuracy (Choice A).
Conclusion:
33
Educational objective:
34 For two iF OBTs, low-dose aspirin use affects performance of the
A shift in the ROC curve upwards for a given cutoff indicates
35 test in detecting advanced colorectal neoplasm.
increased sensitivity. A shift of the curve to the right for a given
36
Funding Source: the German Research Foundation, the German cutoff point indicates a decrease in specificity.
37
38 Federal Ministry of Education and Research. The test kits were
provided free of charge by the manufacturer. Time Spent 40 Copyright© Last updated:
39
40 ~ seconds USMLEWorld ,LLC. [3/7/2013]
2 Item: 22 of 44 11 PMark <J C>- jf ~· 1.~ I

:CJ Q.ld: 7712 [ Previous Ne><t Lab Values Notes Calculator

5
6
7
To assess the association of low-dose aspirin use with the
performance of two quantitative immunochemical fecal occult blood
r The cutoff point for iF OBT recommended by manufacture for
hemoglobin test is 1~gig. How would the performance of the
test be affected by setting the cutoff of the test at 4~g/g for
8 tests (iF OBTs) in a large sample of patients undergoing colorectal patients taking low-dose aspirin?

:[lJ cancer screening

Methods: r A. Negative predictive value would increase [18%]


11
12 r B. Number of false-positive would increase [17%]
Design: Cross-sectional study
13 ., r C. Positive predictive value would increase [56%]
14
Blinding Technician performing iF OBT tests was blinded to r D. Sensitivity of the test would increase [1 0%]
15
colonoscopy results

Setting: 20 gastroenterology practices in southern Germany Explanation: User ld:

Patients: Patients undergoing screening colonoscopy (mean age Both sensitivity and specificity depend on the cutoff value of a
62.1 years) were asked to provide stool samples before the given test. In this example, raising the cutoff value makes it
procedure. Information about medication use was collected using a more difficult to diagnose the condition (colorectal neoplasm),
standardized questionnaire. Specific exclusion criteria included the as more hemoglobin must be present in the stool for the test to
following: visible rectal bleeding or previous positive F OBT result, be positive. By raising the cutoff value, it makes it harder to
24 history of inflammatory bowel disease, colonoscopy in the past 5 obtain a positive test result and easier to obtain a negative
25 years, incomplete colonoscopy, inadequate bowel preparation for result. This will increase the specificity but decrease the
26 colonoscopy, participants with pseudopolyps or histologically sensitivity. Lowering the cutoff value would make it easier to
·1271 undefined polyps at screening colonoscopy, and participants who obtain a positive result and harder to obtain a negative result,
·~ reported regular use of high-dose analgesics. and doing so will increase the sensitivity but decrease the
29 specificity.
30 Intervention: Two automated, enzyme-linked immunosorbent assay
31 (ELISA) based iF OBTs (hemoglobin test and The positive predictive value (PPV) and negative predictive
32 hemoglobin-haptoglobin test) were performed before screening values (NPV) are related to sensitivity and specificity, but are
33 colonoscopy. defined differently. PPV can be thought of as the clinical value
34 of a positive test result in a given population. NPV can likewise
35 Outcome measures: Sensitivity, specificity, positive and negative be considered as the clinical value of a negative test result in a
36 predictive values, and area under receiver operating characteristic given population. The higher the PPV or NPV, the greater the
37 (ROC) curves in detecting advanced colorectal neoplasms with two likelihood that a given positive or negative test result,
38 quantitative iF OBTs using colonoscopy as gold standard. respectively, is actually a correct diagnosis.
39
40 Results:
~ !:':~
• 1
• 2 Item: 22 of 44 !II PMark <J t> jf I

:CJ 5
Q.ld: 7712 [

Results.
Previous Next

Positive Negative
Lab Values Notes Calculator

6 1979 patients (233 regular users of low-dose aspirin and 1746 who condition condition
7 never used low-dose aspirin) were studied. Advanced neoplasms
8
Positive Test True positive False positive
were found in 24 users ( 10.3%) and 181 nonusers ( 10 A%) of
result (TP) (FP)
:[lJ low-dose aspirin.
Negative Test False negative True negative
11 result (FN) (TN)
12 Figure 3. Receiver Operating Characteristic Cu rves
13 for Detecting Advanced Colorectal Neoplasms by
14 Quanti tative lmmunochemical Fecal Occult B lood
15 Test According to Use of Low-Dose Aspi rin Sensitivity =TP I (TP+FN) PPV =TP I (TP+FP)
Specificity =TN I (TN+FP) NPV =TNI(TN+FN)
All parti cipants
The equations for PPV and NPV are defined above, along with
Hemoglobin test the ones for sensitivity and specificity. As the sensitivity
Specificity, % increases, the NPV increases (due to fewer FN). As the
specificity increases, the PPV increases (due to lower FP). It
100 80 60 40 20 0 is important to note that while sensitivity and specificity depend
only upon the characteristics of a given test, PPV and NPV also
24 depend upon the prevalence of the condition in the population
25 being tested. PPV varies directly with prevalence (higher
26 80 prevalence correlates with higher PPV), while NPV varies

·c:]
• 28
....................
inversely with prevalence (higher prevalence corresponds with
lower NPV).
29 .··•·
30 #. 60 ..···· The Receiver Operating Characteristic (ROC) curve in the
31 ~ studies above show that an increase in the cutoff value from 1
.~ ~gig to 4 ~gig decreased the sensitivity of the hemoglobin test
32 ""'
(/) ,/// from 70% to 60% and increased the specificity from 80% to
33 c
34 ~ 40 90%. Similarly, the hemoglobin-haptoglobin test also had a
35 decrease in sensitivity from 60% to 40% and an increase in
36 specificity from 80% to 90% with the same increase in cutoff
.•.· values. This increase in specificity correlates with an increase
37 20 ..··
38 ..·· in the PPV.
....
39
•'
..·· ~ (Choice A) NPV would increase following an increase in
40
~ 1.":~
• 1
• 2 Item: 22 of 44 !II PMark <:1 t> jf I

:CJ 5
Q.ld: 7712 [

Results.
Previous Next

y p g '
Lab Values

depend upon the prevalence of the condition in the population


Notes Calculator

6 1979 patients (233 regular users of low-dose aspirin and 1746 who being tested. PPV varies directly with prevalence (higher
7 never used low-dose aspirin) were studied. Advanced neoplasms prevalence correlates with higher PPV), while NPV varies
8 were found in 24 users ( 10.3%) and 181 nonusers ( 10 A%) of inversely with prevalence (higher prevalence corresponds with

:[lJ low-dose aspirin. lower NPV).

11 The Receiver Operating Characteristic (ROC) curve in the


12 Figure 3 . Receiver Operating Characteristic Cu rves studies above show that an increase in the cutoff value from 1
13 for Detecting Advanced Colorectal Neoplasms by ~gig to 4 ~gig decreased the sensitivity of the hemoglobin test
14 Quanti tative lmmunochemical Fecal Occult B lood from 70% to 60% and increased the specificity from 80% to
15 Test According to Use of Low-Dose Aspirin 90%. Similarly, the hemoglobin-haptoglobin test also had a
decrease in sensitivity from 60% to 40% and an increase in
All participants specificity from 80% to 90% with the same increase in cutoff
values. This increase in specificity correlates with an increase
Hemoglobin test in the PPV.
Specificity, % (Choice A) NPV would increase following an increase in
100 80 60 40 20 0 sensitivity. This example demonstrated a decrease in
sensitivity, which would lead to a corresponding decrease in
24 the NPV.
25
26 ..··· (Choices B and D) The number of false positives would
80 decrease if the cutoff rate were increased, since it would be
·c:]
• 28 ....············.... harder to obtain a positive test result. This would also lead to
decreased sensitivity since the number of false negatives
29 .··•·
30 #. 60 ..···· increases .
31 ~
.~ Educational objective:
32
""
(/) ,/// Changing the cutoff value of a test for a given condition alters
33 c
40 the PPV, NPV, sensitivity, and specificity. Increased specificity
34 rJ3 correlates with increased PPV and increased sensitivity
35 corresponds to increased NPV. PPV and NPV also depend
36 .· upon the condition's prevalence .
•'
37 20 ..··
38 ..··
.... Time Spent 3 Copyright© Last updated:
39
•'
..·· ~ seconds USMLEWorld ,LLC. [2111120 13]
40
1.~
• 1
• 2 Item: 23 of 44 11 PMark -<:J t> jf 9-• I

:CJ 5
Q.ld: <W01 [

A clinical study is conducted to assess the role of a vasopressin antagonist on overall survival in patients with
Previous Ne><t Lab Values Notes Calculator

6
advanced heart failure and hyponatremia. After providing informed consent. eligible hospitalized patients are
7
administered either the new drug or a matching placebo along with the appropriate standard care.
8
Assignment to the 2 treatment arms is done randomly using numbers generated by a computer. This
assignment strategy is most helpful for controlling which of the following?
:[lJ
11 r A. Ascertainment bias [30%]
12 .., r B. Confounding [36%]
13
14 r C. Effect modification [11 %]
15 r D. Recall bias [2%]
16 r E. Selective survival [22%]
17
·liBl
·~ Explanation: User ld:
20
Methods to control confounding

24 Design stage
25
Matching
26
·1271 Restriction
·~ Rand om ization
29
30 Analysis stage
31
St ratified analysis
32
33 Statistical modelin g
34
€1 USMLEWOfld, UC
35
36
37 The effects of confounding bias can be reduced through good study design and proper analysis of the results.
38 Methods used to control confounding during data analysis include stratified analysis and statistical modeling
39 (eg. multivariate analyses). Methods to control confounders during study design include the following:
40 1 t.ll ......... t.-.: ......... r .......................... ,,...,......J ; ....................................,..., ....... ,...J;,...,... ..~.-.; .... __... ......t.-. .... ...J ; ...., ,,...,, ,,...,... __... .......... t.-.: ........ 1, ......... ,. ................ ....................... ...J
2 Item: 23 of 44 !il f>Mark <:J C>- jf ~· 1.":~ I

:CJ
5
Q.ld: <W01 [ Previous Ne><t

The effects of confounding bias can be reduced through good study design and proper analysis of the results.
Lab Values Notes Calculator

6 Methods used to control confounding during data analysis include stratified analysis and statistical modeling
7 (eg. multivariate analyses). Methods to control confounders during study design include the following:
8 1. Matching: Frequently used in case-control studies. this method involves matching known or suspected
:[lJ confounding variables between the cases and controls. For example. if smoking status is a known
confounder. then controls will be selected on a case by case basis so that there are similar numbers
11 of smokers and nonsmokers in both the case and control groups.
12 2. Restriction: This method involves limiting study participation to individuals with specific
13 characteristics. For example. if sex is thoughtto be a confounder. then a restricted study may enroll
14 only men. However. this method is limited because the study results may not be applicable to the
15 excluded population (ie. women).
16 3. Randomization: Commonly employed in clinical trials. this technique helps to balance the distribution
17 of confounding variables between treatment and placebo groups so that the unconfounded effect of
·liBl the exposure of interest can be isolated. An important advantage of randomization. compared to other
·~ methods. is the possibility of controlling known risk factors (eg. age & severity of heart failure) as well
20 as the unknown and difficult-to-measure confounders (eg. level of stress & socioeconomic status).

Randomization also eliminates partiality in treatment assignments (minimizing selection bias) and blinds
24 investigators to the identity of patients who receive the treatment arm (reducing observer bias).
25
(Choice A) Ascertainment (sampling) bias occurs due to nonrandom sampling of a target population. This
26
causes the characteristics of the study population to differ from those of the target population. leading to
·1271 results that may not be generalizable.
·~
29 (Choice C) Effect modification results when an external variable positively or negatively impacts the effect of
30 a risk factor on the disease of interest. For instance. the risk of venous thrombosis is increased with estrogen
31 therapy. and this effect is augmented by smoking.
32
33 (Choice D) Recall bias results from inaccurate recall of past exposure and applies primarily to retrospective
34 studies. People who have suffered an adverse event are more likely to recall risk factors than people without
35 such experiences.
36
37 (Choice E) Selective survival bias occurs in case-control studies when cases are selected from the entire
38 disease population instead of just those that are newly diagnosed. For instance. a study on cancer survival
39 that is not limited to newly diagnosed patients will contain a higher proportion of relatively benign malignancies
40 :::.~ thP~P n:::.tiPnt~ nPnPr:::.llv livP lnnnPr
2 Item: 23 of 44 !il f>Mark <:J C>- jf ~· 1.":~ I

:CJ
5
Q.ld: <W01 [ Previous Ne><t
contounaer. men controls will be selectea on a case by case basis so mat mere are s1m11ar numbers
of smokers and nonsmokers in both the case and control groups.
Lab Values Notes Calculator

6 2. Restriction: This method involves limiting study participation to individuals with specific
7 characteristics. For example. if sex is thought to be a confounder. then a restricted study may enroll
8 only men. However. this method is limited because the study results may not be applicable to the
excluded population (ie. women).
:[lJ 3. Randomization: Commonly employed in clinical trials. this technique helps to balance the distribution
of confounding variables between treatment and placebo groups so that the unconfounded effect of
11
12 the exposure of interest can be isolated. An important advantage of randomization. compared to other
13 methods. is the possibility of controlling known risk factors (eg. age & severity of heart failure) as well
14 as the unknown and difficult-to-measure confounders (eg. level of stress & socioeconomic status).
15
16 Randomization also eliminates partiality in treatment assignments (minimizing selection bias) and blinds
17 investigators to the identity of patients who receive the treatment arm (reducing observer bias).
·liBl
·~ (Choice A) Ascertainment (sampling) bias occurs due to nonrandom sampling of a target population. This
20 causes the characteristics of the study population to differ from those of the target population. leading to
results that may not be generalizable.

24
(Choice C) Effect modification results when an external variable positively or negatively impacts the effect of
a risk factor on the disease of interest. For instance. the risk of venous thrombosis is increased with estrogen r
25 therapy. and this effect is augmented by smoking.
26
(Choice D) Recall bias results from inaccurate recall of past exposure and applies primarily to retrospective
·1271 studies. People who have suffered an adverse event are more likely to recall risk factors than people without
·~ such experiences.
29
30
(Choice E) Selective survival bias occurs in case-control studies when cases are selected from the entire
31
disease population instead of just those that are newly diagnosed. For instance. a study on cancer survival
32
that is not limited to newly diagnosed patients will contain a higher proportion of relatively benign malignancies
33
as these patients generally live longer.
34
35 Educational objective:
36 Randomization is used to control confounders during the design stage of a study. It helps to control known
37 confounders as well as unknown and difficult-to-measure confounders.
38
39
Time Spent 2 seconds Copyright© USMLEWorld .LLC. Last updated [7/12/20 13]
40
~· 1.~~
• 1
• 2 Item: 24 of 44 !II f> Mark -<J !>- jf I

:CJ 5
Q.ld: 4189 [ Previous Ne><t
A 40-year-old woman presents to the office with several months history of atypical chest pain. Her family
history is positive for hypertension and type II diabetes mellitus. Her past medical history is insignificant. An
Lab Values Notes Calculator

6 ECG stress test is performed to evaluate the possibility of coronary heart disease. This test has 85%
7 sensitivity and 80% specificity. In the low-risk group. its positive predictive value is 25%. and negative
8 predictive value is 96%. If the test is negative. what is the probability that the patient has coronary heart

:[lJ disease?
r A. 25% [4%]
11
12 r B. 20% [8%]
13 r c. 15% [10%]
14
.., r D. 4% [75%]
15
16 r E. 1% [2%]
17
·liBl Explanation: User ld:
·~
• 20 Pre- and post-test probabilities. as well as positive and negative predictive values. are very important
·1211 concepts in clinical medicine. as they dramatically change the approach to the diagnosis of each disease .
. LBJ Take the time to learn and fully understand these terms.
23
The pre-test probability sets your expectations before performing the diagnostic test. It describes the existing
probability of a patient to have the disease in question even before using a particular diagnostic test. (Although
26 there is a mathematical approach to calculate the pre-test probability. we shall simplify this term and use the
·1271 most basic explanations.) Using the example given above. the pre-test probability of the patient having
·~ coronary artery disease can be assumed to be low since she has no risk factors aside from her significant
29 family history. and since she has atypical chest pain. Statistically. the prevalence is directly related to the
30 pre-test probability. In this example. the patient belongs to the low-risk group. which consistently has a
31 relatively low prevalence of coronary artery disease.
32
33 The positive predictive value (PPV) describes the probability of having the disease if the test result is positive.
34 The post-test probability of having the disease is directly related to the PPV. Using the example above. the
35 PPV (25%) is low. Consequently. if the test result is positive. then the post-test probability of having the
36 disease is low. The post-test probability is also dependent on the sensitivity. specificity. and pre-test
37 probability of having the disease.
38
39 The negative predictive value (NPV) describes the probability of not having the disease if the test result is
40 neqative. In this example. the NPV is 96%. This means that if the test result is neqative. the chances of the
~· 1.":~
• 1
• 2 Item: 24 of 44 !il f> Mark <:J C>- jf I

:CJ 5
Q.ld: 4189 [ Previous Ne><t

Pre- and post-test probabilities. as well as positive and negative predictive values. are very important
Lab Values Notes Calculator

concepts in clinical medicine. as they dramatically change the approach to the diagnosis of each disease.
6
Take the time to learn and fully understand these terms.
7
8 The pre-test probability sets your expectations before performing the diagnostic test. It describes the existing
:[lJ probability of a patient to have the disease in question even before using a particular diagnostic test. (Although
there is a mathematical approach to calculate the pre-test probability. we shall simplify this term and use the
11 most basic explanations.) Using the example given above. the pre-test probability of the patient having
12
coronary artery disease can be assumed to be low since she has no risk factors aside from her significant
13 family history. and since she has atypical chest pain. Statistically. the prevalence is directly related to the
14
pre-test probability. In this example. the patient belongs to the low-risk group. which consistently has a
15 relatively low prevalence of coronary artery disease.
16
17 The positive predictive value (PPV) describes the probability of having the disease if the test result is positive.
·liBl The post-test probability of having the disease is directly related to the PPV. Using the example above. the
·~ PPV (25%) is low. Consequently. if the test result is positive. then the post-test probability of having the
• 20 disease is low. The post-test probability is also dependent on the sensitivity. specificity. and pre-test
·1211 probability of having the disease .
. LBJ
23 The negative predictive value (NPV) describes the probability of not having the disease if the test result is
negative. In this example. the NPV is 96%. This means that if the test result is negative. the chances of the
patient to not have the disease is high (96%). Conversely. if the test result is negative. the chances of the
26 patientto have the disease is low ( 100% - 96%= 4%).
·1271
·~ Other cases and diagnostic tests which are high-yield for the USMLE are listed below:
29 1. Pulmonary embolism and perfusion-ventilation scanning
30
2. Prostate cancer and serum PSA levels
31
32
33 (Choice B) False positive ratio= 1- specificity.
34
35
(Choice C) False negative ratio= 1 -sensitivity.
36
Educational Objective:
37
If a test result is negative. the probability of having the disease is 1 - negative predictive value.
38
39
Time Spent 3 seconds Copyright© USMLEWorld .LLC. Last updated [7/7/20 10]
40
1.~
• 1
• 2 Item: 25 of 44 1111 PMark <J C>- jf 9-• I

:CJ 5
Q.ld: 3653 [ Previous Ne><t Lab Values Notes Calculator

6 A study was conducted to evaluate the efficacy of a new antiviral drug for the treatment of the common cold in
7 young children. The study population consisted of 1DO children between the age of 2 to 8 years. These
8 children were diagnosed with rhinovirus infection and subsequently given the particular antiviral drug. One

:[lJ week later. it was observed that 92 of the 1DO patients were asymptomatic. Which of the following is the true
conclusion of this study?
11
12
13
r A. The drug is highly effective as the effectiveness is 90% [2%]
14 r B. The drug is moderately effective as the efficacy is 90% [1 %]
15 ., r C. An exact conclusion cannot be drawn from the study. as common cold is often a self-limiting
16 disease of less than 1 week [93%]
17
r D. The drug is not effective as the sample size is very small [2%]
·liBl r E. No conclusion can be made. as compliance is generally very low in small children [1 %]
·~
• 20
·1211
. LBJ Explanation: User ld:
23
It is very importantto consider the natural history of a disease when evaluating any drug trial. The common
24
cold is a self-limiting disease. which generally resolves within a week. For this reason. conclusions are
difficult to draw regarding the effectiveness of the drug. The resolution of symptoms in the 90% of the study
26
population may be due to natural resolution rather than the drug. For such drug trials. a control group is useful
·1271 since it helps exclude the bias due to natural resolution of the disease.
·~
29 (Choices A, 8 and D) It is difficult to comment on the effectiveness of the drug. unless a comparison is made
30 with the control group and statistical significance is calculated to know the power of the study.
31
32 (Choice E) The reason why no conclusion can be made is the absence of a control group in the study design.
33 rather than improper patient compliance. Nevertheless. it is importantto consider the level of compliance in a
34 drug study done on small children.
35
36 Educational objective:
37 Consider the natural history of a disease when evaluating the effectiveness of a drug in a trial.
38
39 Time Spent 1 seconds Copyright© USMLEWorld .LLC. Last updated [7/7/20 10]
40
2 Item: 26 of 44 1111 PMark <J C>- jf 9-• 1.~ I

:CJ
5
Q.ld: 3989 [ Previous Ne><t

A study was conducted to assess the relationship between serum HDL2 level (a subtraction of HDL) and
Lab Values Notes Calculator

6
carotid intima-media thickness as a marker of atherosclerosis. Interpretation of the results revealed a linear
7
8
= =
relationship between these two variables and the correlation coefficient (r -0.35. p 0.005). Which of the
following statements about the results of the study is the most correct?
:[lJ r A. The association is not statistically significant [5%]
11 r B. There is a strong correlation between HDL2 level and carotid intima-media thickness [17%]
12
., r C. As the level of HDL2 increases the carotid intima-media thickness decreases [65%]
13
14 r D. There is positive correlation between HDL2 level and carotid intima-media thickness [9%]
15 r E. Decreased HDL2 level is the cause of carotid intima-media thickening [4%]
16
17
·liBl Explanation: User ld:
·~ The correlation coefficient assesses a linear relationship between two variables. The null value for the
• 20
correlation coefficient is 0 (no association). and the range of plausible values is from -1 to 1. The sign of the
·1211
. LBJ correlation coefficient indicates a positive or negative association. The closer the value is to its margins (-1 or
1). the stronger the association.
23
(Choice A) The association is actually statistically significant because the p value is low.

(Choice B) The strength of association is weak because 0.35 is close to 0.

(Choice D) In this scenario. 'r' is negative. This means that as the level of HDL2 increases. the carotid
intima-media thickness decreases.
30
31 (Choice E) The correlation coefficient shows the strength of association. but does NOT necessarily imply
32 causality.
33
34 Educational Objective:
35 The correlation coefficient shows the strength and direction (positive. negative) of linear association between
36 two variables. It does not necessarily imply causality.
37
38 Time Spent 2 seconds Copyright© USMLEWorld .LLC. Last updated [7/7/20 10]
39
40
~· 1.~
• 1
• 2 Item: 27 of 44 11 f>Mark -<:1 C>- jf I

:CJ 5
Q.ld: 7688 [

Risk of Hyperkalemia in Non-diabetic Patients with Chronic


Previous Ne><t Lab Values Notes Calculator

6 Kidney Disease Receiving Antihypertensive Therapy A 43-year-old African American male with a strong family
7 history of hypertension presents to the office with generalized
8 Objective: fatigue. His past medical history is noncontributory. He is an
active smoker. He is currently taking metoprolol. His blood
:[lJ Explore the incidence and factors associated with hyperkalemia in
patients with chronic kidney disease (CKD) treated with
pressure is 162/1 DO mmHg and his pulse is 76/min. The apical
impulse is laterally displaced. His physical examination is
11
12 antihypertensive drugs otherwise unremarkable. EKG shows high voltage consistent
13 with left ventricular hypertrophy with secondary repolarization
Methods: changes. His serum potassium is 3.9 mEq/L and estimated
14
15 GFR is 36 mUmin/1.73 m2 . His baseline UP/Cr is 0.20. The
Design: Randomized clinical trial
16 patient is started on an ACE inhibitor and he shows compliance
17 Blinding Double-blinded with follow-up visits. Based on study results. which of the
·liBl following factors on follow-up will most significantly increase his
·~ Follow-up: 3 to 6.4 years chance of hyperkalemia event?
• 20
·1211 Setting Multicenter (21 medical centers) r A. Addition of a calcium channel blocker [7%]
. LBJ Patients: African American patients. aged 18 to 70 years. with r B. Addition of a thiazide diuretic [8%]
23
24 hypertensive CKD as defined by a diastolic blood pressure (BP) r C. Discontinuation of metoprolol [5%]
25 higher than 95 mm Hg and a glomerular filtration rate (GFR)
between 20 and 65 mUminl1.73 m2 . Specific exclusion criteria
., r D. Follow-up serum potassium level of 4.7 meq/1 as
26 compared to baseline [70%]
included diabetes mellitus; urinary protein to urinary creatinine ratio
(UP/Cr) higher than 2.5; accelerated or malignant hypertension; r E. Fallow-up UP/Cr of 1.1 as compared to baseline
secondary hypertension; serious systemic disease; congestive [10%]
29
30
heart failure; and initial potassium level higher than 5.5 mEq/L.
31 Explanation: User ld:
Intervention: Patients were randomized to initial treatment with
32
either a beta-blocker (metoprolol). an ACE inhibitor (ramipril). or a
33
calcium channel blocker (amlodipine) and to 1 of 2 mean arterial BP This study is a double-blinded randomized clinical trial
34 comparing the incidence of hyperkalemia in African-American
goals ( 102-1 07 mmHg or <92 mmHg).
35 (non-diabetic) patients with CKD and hypertension (diastolic
36 Outcome measures: hyperkalemic event (potassium level higher greater than 95 mm Hg). The patients were randomized to
37 than 5.5 mEq/L). either metoprolol. amlodipine. or ramipril and followed up to 6
38 years. The primary endpoint was the incidence of
39 Results: · ' hyperkalemia. defined as greater than 5.5 mEq/L. The results
\AtPrP rPnnrtPrl :=I~ P\/Pnt~ nPr 1nn n:::.tiPnt \/P:::Ir~ :::.nrl :::.l~n
1

40 :..:..J
~· ~
• 1
• 2 Item: 27 of 44 !il PMark <:J t:> jf I

:CJ 5
Q.ld: 7688 [

Results:
Previous Ne><t

reported as hazard ratios.


Lab Values Notes Calculator

6
A total of 1094 non-diabetic patients were randomized. A total of Hazard ratio is the ratio of an event rate occurring in the
7
6497 potassium measurements were obtained. and 80 treatment group compared to an event rate occurring in the non-
8
hyperkalemic events in 51 subjects were identified. treatment group. A ratio less than 1 indicates that the
:[lJ treatment group had a significantly lower event rate while
values greater than 1 indicate that the treatment group had a
11
12 ~ 15 RaMomlzed Oruo Groop much higher event rate. In this study. the risk factors for
0 Rarnipril developing hyperkalemia were reported as hazard ratios. The
13 ~- 12 0 Metroprol succinata higher the ratio. the more likely that factor is associated with the
14 • AmiOPidine besylale
15
E incidence of hyperkalemia. The factor that had the highest ratio
16
·"'&! 9 (of the choices listed) is a follow-up serum potassium level
17
0
s;! between 4-5 mEq/L as compared to less than 4 mEq/L. with a
6 hazard ratio of 7.25.
·liBl ~
·~
• 20
.!l
"
a:
E
3 (Choice A) As shown in the results figure. patients with a GFR
between 30-40 mUmin/1 .73 m2 given amlodipine actually had a
r
·1211 !!!
w lower incidence of hyperkalemia per 1DO patient years than
. LBJ 0
ramipril.
23 Baseline GFR
24
(Choice B) The hazard ratio for follow-up use of a diuretic was
25 0 A 1 (with a statistically significant p value <0 .01). indicating
figure 1. Hyperkalemia event rate per 100 patient-years by randomized drug
26
groups and baseline glomerular filtration rate (GFR). Error bars indicated that adding a diuretic is associated with a lower incidence of
95% confidence intervals. hyperkalemia.
29 (Choice C) This patient is already on metoprolol and has a
30 Table 4. Association of Risk of Hyperkalemia With normal potassium. The addition of metoprolol in the study did
31 Time-Dependent Factors In Mulllvariable Analysls 3 not significantly increase the incidence of hyperkalemia in
32 patients with GFR > 30 and ~40 according to the figure above.
33 so discontinuation of the drug likely would not have a significant
Hazard Ratio
34 p effect on the potassium.
(95% Conlidence
35
Variable Interval) Value
36 (Choice E) Fallow-up UP/Cr of 1.1 as compared to baseline
37 Follow-up diuretic use 0.41 (0.22-0.78) .006 has a hazard ratio of 1.84. indicating that there is an increased
38 follow-up GFR s 30 vs 9.07 (3.1 8-25.88) <.001 incidence of hyperkalemia. However. this ratio is lower than the
39 > 50 mUmin/1.73 m2 hazard ratio for follow-up potassium level between 4-5 mEq/L
40 Follow-uo GFR > 30 to_;;s40 vs 3.6711 .21-11.15\ .02_ ~ ;:~nrl it i~ nnt ~t;:~ti~tir.;:~llv ~innifir.;:~nt (n=n 1R\
It~
• 1
• 2 Item: 27 of 44
Q.ld : 7688 [
11 f> Mark -<:1 1>- il I'Ol:J""

I

:CJ 5
I

Time-Dependent Factors In Multlvariable Analysls 3


Prevrous Ne><t Lab Values Notes Calculator

6 (Choice B) The hazard ratio for follow-up use of a diuretic was


7 0 A 1 (with a statistically significant p value <0 .01). indicating
Hazard Ratio that adding a diuretic is associated with a lower incidence of
8 p
(95% Confidence hyperkalemia.
:[lJ Variable
Follow-up diuretic use
Interval)
0.41 (0.22-0.78)
Value
.006 (Choice C) This patient is already on metoprolol and has a
11
12 Follow-up GFR :!i30 vs 9.07 (3.1 8-25.88) <.001 normal potassium. The addition of metoprolol in the study did
> 50 mUmin/1.73 m2 not significantly increase the incidence of hyperkalemia in
13
Follow-up GFR > 30 to :s40 vs 3.67 (1 .21-1 1.15) .02 patients with GFR >30 and ~40 according to the figure above.
14
> 50 mUmln/1.73 mz so discontinuation of the drug likely would not have a significant
15
16 Follow-up GFR > 40 to :s50 vs 1.98 (0.59-6.61) .27 effect on the potassium.
17
> 50 mUmin/1.73 mz
Follow-up UP/Cr > 0.08 to :s0.22 2.01 (0.92-4.39) .08 (Choice E) Fallow-up UP/Cr of 1.1 as compared to baseline
·liBl vs !50.08 has a hazard ratio of 1.84. indicating that there is an increased
·~ Follow-up UP/Cr >0.22 to :s0.66 1.50 (0.62-3.63) .37 incidence of hyperkalemia. However. this ratio is lower than the
• 20
vs !50.08 hazard ratio for follow-up potassium level between 4-5 mEq/L
·1211 Follow·up UP/Cr > 0.66 vs :s0.08 1.84 (0.78-4.30) .16 and it is not statistically significant (p=O .16).
. LBJ follow·up potassium level4-5 7.25 (1.72·30.58) .007
23 Educational objective:
24 vs < 4 mEq/L
Follow-up potassium level > 5 30.83 (6.89-138.0) < .001 Hazard ratios are the ratio of an event rate occurring in the
25 treatment arm versus the non-treatment arm. Ratios less than
vs < 4 mEq/L
26 1 indicate that the treatment arm had a lower event rate while
ratios higher than one indicate the treatment arm had a higher
Conclusion: rate of events.
29
30 In non-diabetic patients with hypertensive CKD treated with ACEis. References:
31 the risk of hyperkalemia is small.
32 1. Biostatistics primer: what a clinician ought to
33 Funding Source: The National Institute of Diabetes and Digestive know: hazard ratios.
34 and Kidney Diseases (NIDDK) grant; additional financial support 2. Estimation of the 2 -sample hazard ratio
35 from the Office of Research in Minority Health and drug donations function using a semi parametric model.
36 from Pfizer Inc. AstraZeneca Pharmaceuticals. and King
37 Pharmaceuticals.
38
Structured abstract is based on: Arch Intern Med. Time Spent 18 Copyright© Last updated:
39
40 2009; 169( 17) 1587-94 ~ seconds USMLEWorld .LLC. [7/21/2013]


1
2 Item: 28 of 44
O.ld: 7689 (
• I ' Ma•k <:::]
·
[:> jf ~·

l~:'ll

:CJ 5
Prev•ous NeHt

For the study, patients were randomized to initial treatment with


Lab Values Notes Calculato1·

6 Risk of Hyperkalemia In Non-diabetic Patients with Chronic either a beta-blocker (metoprolol), an ACE inhibitor (ramipril), or
7 Kidney Disease Receiving Antihypertensive Therapy a calcium channel blocker (amlodipine) and to 1 of 2 mean
8 arterial BP goals ( 102-1 07 mmHg or <92 mmHg). The study is
best classified as:
:[lJ Objective:

Explore the incidence and factors associated with hyperkalemia in


11 r A. Cluster (12%)
12 patients with chronic kidney disease (CKD) treated with
13 antihypertensive drugs r B . Cross-over (12%)
14 >~ r C . Factorial des1gn [28%)
Methods:
15 r D . Parallel-group [48%)
16
Design: Randomized clinical trial
17

:I::I
• 20
Blinding: Double-blinded

Follow-up: 3 to 6.4 years


Explanation :

Factorial Design Study Exhibit


User ld:

:I ~~I
23
Selling: Multicenter (21 medical centers) This study utili zes 3 different interventions (metoprolol, ramipril.
amlodipine) with 2 different variable blood pressure endpoints
Patients: African American patients . aged 18 to 70 years. with (1 02-107 mm Hg or <92 mm Hg), which is consistent with a
hypertensive CKD as defined by a diastolic blood pressure (BP) factorial design study. A factorial design (as shown above)
higher than 95 mm Hg and a glomerular filtration rate (GFR) involves 2 or more experimental interventions. each with 2 or
between 20 and 65 mUmin/1.73 m2 . Specific exclusion criteria more variables that are studied independently. In this study, the
included diabetes mellitus: urinary protein to urinary creatinine ratio patients w ere being investigated to see which one of these 3
(UP/Cr) higher than 2.5: accelerated or malignant hypertension: drugs caused hyperkalemia with the greatest frequency. In
29
secondary hypertension: serious systemic disease: congestive addition. the drugs also had 2 different blood pressure end
30
heart failure: and initial potassium level higher than 5.5 mEq/L points rather than one common end point to see what effect
31
they also had on blood pressure.
32
33
Intervention: Patients were randomized to initial treatment with
either a beta-blocker (metoprolol), an ACE inhibitor (ramipril), or a (Choice A) Cluster analysis is the grouping of different data
34
calcium channel blocker (amlodipine) and to 1 of 2 mean arterial BP point into similar categories . which is not employed in this
35
goals (1 02- 107 mmHg or <92 mmHg). study. Cluster analysis usually involves randomization at the
36
level of groups rather than at the level of individuals.
37
Outcome measures: hyperkalemic event (potassium level higher
38 (Choice B) A cross-over study is one in which group of
than 5.5 mEqll).
39 .:::J participants is randomized to one treatment for a period of time
40 ....... ...t ...................................... : ........ • ........ .... ,.......................................................... f ........... ....
• 1
• 2

:CJ 5
A total of 1094 non-diabetic patients were randomized. A total of addition, the drugs also had 2 different blood pressure end
points rather than one common end point to see what effect
6497 potassium measurements were obtained, and 80
6 they also had on blood pressure.
hyperkalemic events in 51 subjects were identified.
7
8 (Choice A) Cluster analysis is the grouping of different data

:[lJ ~ 15
l!i
RaMomlzed Oruo Groop
0 Rarniprll
point into similar categories, which is not employed in this
study. Cluster analysis usually involves randomization at the
11 0 Metroprol succinate level of groups rather than at the level of individuals.
12 ~
E
12
• AmiOPidine besylale
13 (Choice B) A cross-over study is one in which group of
14
·"'&'! 9
participants is randomized to one treatment for a period of time
0
15 s;! and the other group is given an alternate treatment for the
6
16 ~ same period of time. At the end of the time period, the two
17 .!l groups then switch treatments for another set period of time .
·liBl "'
a: 3
This study gave the same treatment to the patients for the
E
·~ w
~
0
duration of the trial.
• 20
(Choice D) A parallel study randomizes one treatment to one
·1211
. LBJ group and a different treatment to the other group, such as
treatment drug to one group versus placebo to the other group.
23
Figure 1. Hyperkalemia event rate per 100 patient·years by randomized drug There are usually no other variables measured, such as the
24
groups and baseline glomerular filtration rate (GFR). Error bars indicated two blood pressure goals in this study.
25 95% confidence intervals.
Educational objective:
Factorial design studies involve randomization to different
Table 4. Association of Risk of Hyperkalemia With interventions with additional study of 2 or more variables.
Time-Dependent Factors In Mulllvariable Analysls 3
30
31 References:
Hazard Ratio
32 p 1. Design of experiments with multiple
(95% Confidence
33 Variable Interval) Value independent variables: a resource
34 management perspective on complete and
35 Follow-up diuretic use 0.41 (0.22-0.78) .006
reduced factorial designs.
36 Follow·up GFR s 30 vs 9.07 (3.1 8-25.88) <.001
37 > 50 mUmin/1.73 m2
38 Follow·up GFR > 30 to :s40 vs 3.67 (1.21-1 1.15) .02
> 50 mUmln/1.73 mz Time Spent 8 Copyright© Last updated:
39
Follow-up GFR > 40 to s so vs 1.98 (0.59·6.61) .27 ~ seconds USMLEWorld ,LLC. [7/20/2013]
40
~~~
• 1
• 2 Item: 28 of 44
Q.ld : 7689 [
11 !" Mark -<:1 t:> M.f rot&•
~
I

Lab -Values ·"

:CJ 5
. I . I • I ' T. • • I .•
Prevrous Ne><t

n. the drugs also had 2 different blood pressure end


points rather than one common end point to see what effect
Notes Calculator

6 Hazard Ratio they also had on blood pressure.


7 (95% Confidence p
8 Variable Interval) Value (Choice A) Cluster analysis is the grouping of different data

:[lJ Follow-up diuretic use


follow-up GFR s 30 vs
0.41 (0.22-0.78)
9.07 (3.1 8-25.88)
.006
<.001
point into similar categories. which is not employed in this
study. Cluster analysis usually involves randomization at the
11
>50 mUmin/1.73 m2 level of groups rather than at the level of individuals.
12
Follow-up GFR >30 to :s40 vs 3.67 (1.21 -11.15) .02
13 (Choice B) A cross-over study is one in which group of
> 50 mUmln/1.73 mz
14 participants is randomized to one treatment for a period of time
Follow-up GFR > 40 to :sSO vs 1.98 (0.59-6.61) .27
15 >50 mUmin/1.73 mz and the other group is given an alternate treatment for the
16 same period of time. At the end of the time period. the two
Follow-up UP/Cr > 0.08 to :s0.22 2.01 (0.92·4.39) .08
17 vs :50.08 groups then switch treatments for another set period of time.
·liBl Follow-up UP/Cr > 0.22 to s 0.66 1.50 (0.62·3.63) .37
This study gave the same treatment to the patients for the
-~ vs :50.08 duration of the trial.
• 20
·1211
Follow·up UP/Cr > 0.66 vs :s0.08
follow·up potassium level4·5
1.84 (0.78-4.30)
7.25 (1.72·30.58)
.16
.007
(Choice D) A parallel study randomizes one treatment to one r
. LBJ vs < 4 mEqll
group and a different treatment to the other group. such as
treatment drug to one group versus placebo to the other group.
23 Follow-up potassium level > 5 30.83 (6.89-138.0) <.001
24 There are usually no other variables measured. such as the
vs < 4 mEqll
25 two blood pressure goals in this study.

Conclusion: Educational objective:


Factorial design studies involve randomization to different
In non-diabetic patients with hypertensive CKD treated with ACEis. interventions with additional study of 2 or more variables.
30 the risk of hyperkalemia is small.
31 References:
32 Funding Source: The National Institute of Diabetes and Digestive
and Kidney Diseases (NIDDK) grant; additional financial support 1. Design of experiments with multiple
33 independent variables: a resource
34 from the Office of Research in Minority Health and drug donations
from Pfizer Inc. AstraZeneca Pharmaceuticals. and King management perspective on complete and
35 reduced factorial designs.
36 Pharmaceuticals.
37
Structured abstract is based on: Arch Intern Med.
38
2009; 169( 17) 1587-94 Time Spent 8 Copyright© Last updated:
39
40
• seconds USMLEWorld .LLC. [7/20/2013]
2 Item: 29 of 44 11 PMark <:J 1>- jf 9-• 1.":~ I

:CJ
5
Q.ld: 364S [ Previous Ne><t Lab Values Notes Calculator

6 A clinical trial is performed to study the effect of a new antihypertensive drug. If the subjects of the study
7 change their behavior because they are aware that they are under observation. what type of bias will take
8 place?

:[lJ r A. Sample distortion bias [7%]


11
12 r B. Information bias [14%]
13 r C. Confounding bias [6%]
14
15
., r D. Hawthorne effect [69%]
16 r E. Not a source of bias [3%]
17
·liBl Explanation: User ld:
·~
• 20
Bias poses a threat to the validity of any study. A sample which is not representative of the population is said
·1211 to be a biased sample. In this vignette. a biased sample with Hawthorne effect is described .
. LBJ
23 Hawthorne effect can be defined as the tendency of a study population to affect the outcome because these
24 people are aware that they are being studied. This awareness leads to a consequent change in behavior
25 while under observation. thereby seriously affecting the validity of the study. Hawthorne effect is commonly
26 seen in studies that concern behavioral outcomes or outcomes that can be influenced by behavioral
changes. In order to minimize the potential of the Hawthorne effect. studied subjects can be kept unaware
that they are being studied; however. this may pose ethical problems. Randomized control trials have a
sense of uncertainty and risk due to randomization. which may be more potent behavior modifiers than mere
30 observation.
31
32 (Choice A) Sample distortion bias is seen when the estimate of exposure and outcome association is biased
33 because the study sample is not representative of the target population with respect to the joint distribution of
34 exposure and outcome.
35
36 (Choice B) Information bias occurs due to the imperfect assessment of association between the exposure
37 and outcome as a result of errors in the measurement of exposure and outcome status. It can be minimized
38 by using standardized techniques for surveillance and measurement of outcomes. as well as trained
39 observers to measure the exposure and outcome.
40
2 Item: 29 of 44 11 PMark <:1 1>- jf 9'' 1.":~ I

:CJ
5
Q.ld: 364S [

., r D. Hawthorne effect [69%]


Previous Ne><t Lab Values Notes Calculator

6
r E. Not a source of bias [3%]
7
8 Explanation: User ld:
:[lJ Bias poses a threat to the validity of any study. A sarnple which is not representative of the population is said
11 to be a biased sarnple. In this vignette. a biased sarnple with Hawthorne effect is described.
12
13 Hawthorne effect can be defined as the tendency of a study population to affect the outcome because these
14 people are aware that they are being studied. This awareness leads to a consequent change in behavior
15 while under observation. thereby seriously affecting the validity of the study. Hawthorne effect is cornrnonly
16 seen in studies that concern behavioral outcomes or outcomes that can be influenced by behavioral
17 changes. In order to rninirnize the potential of the Hawthorne effect. studied subjects can be kept unaware
·liBl that they are being studied; however. this rnay pose ethical problems. Randomized control trials have a
·~ sense of uncertainty and risk due to randomization. which rnay be rnore potent behavior modifiers than rnere
• 20 observation.
·1211
. LBJ (Choice A) Sarnple distortion bias is seen when the estimate of exposure and outcome association is biased
23 because the study sarnple is not representative of the target population with respect to the joint distribution of
24 exposure and outcome.
25
26 (Choice B) Information bias occurs due to the imperfect assessment of association between the exposure
and outcome as a result of errors in the rneasurernent of exposure and outcome status. It can be rninirnized
by using standardized techniques for surveillance and rneasurernent of outcomes. as well as trained
observers to measure the exposure and outcome.
30
(Choice C) Confounding bias occurs due to the presence of one or rnore variables associated independently
31
with both the exposure and the outcome. For example. cigarette smoking can be a confounding factor in
32
studying the association between maternal alcohol drinking and low birth weight babies. as cigarette smoking
33
is independently associated with both alcohol consumption and low birth weight babies.
34
35 Educational objective:
36 Know the different kinds of bias. which can decrease the validity of study results. Hawthorne effect is the
37 tendency of the study population to affect the outcome since they are aware that they are being studied.
38
39
Tirne Spent 2 seconds Copyright© USMLEWorld .LLC. Last updated [7/7/20 10]
40
1.":~
• 1
• 2 Item: 30 of 44 11 PMark <:J 1>- jf 9-• I

:CJ 5
Q.ld: 3932 [ Previous Ne><t

A new test is devised to measure serum cholesterol level. A sample of blood is taken from a patient. and the
Lab Values Notes Calculator

6
test is performed three times. The results are 200. 190 and 184 mg/dl. Based on these results. one can
7 conclude that the new test is·
8
., r A. Not reliable [62%]
:[lJ r B. Not valid [4%]
11 r C. Not accurate [28%]
12
13
r D. Not sensitive [3%]
14 r E. Not specific [3%]
15
16
17 Explanation: User ld:
·liBl
·~ This example deals with test-retest reliability. A reliable test gives similar or very close results on repeat
• 20 measurements. In this example. repeat measurements of the same sample yielded different results;
·1211 therefore. the new test is not reliable .
. LBJ (Choices B and C) Validity or accuracy is defined as the test's ability to measure what it is supposed to
23
24 measure. In order to determine the validity of a test. the results are compared to those obtained from the gold
25 standard test. In this case. since there were no test results obtained using the gold standard. the validity or
accuracy of the test cannot be determined.

(Choices D and E) The sensitivity and specificity of a test compare results to those obtained using the gold
standard. These parameters can show how accurate the results are. but do not measure reliability.

Educational objective:
A reliable test gives similar results on repeat measurements. Reliability is maximal when random error is
32
minimal.
33
34
Time Spent 2 seconds Copyright© USMLEWorld .LLC. Last updated [7/7/20 10]
35
36
37
38
39
40
2 Item: 31 of 44 11 PMark -<J C>- jf ""• 1.":~ I

:CJ
5
Q.ld: 4121 [ Previous Ne><t

A large-scale randomized. double-blinded clinical trial was conducted to evaluate the effect of beta-blocker
Lab Values Notes Calculator

6
therapy on the survival of patients with chronic heart failure. class IV. The patients with severe heart failure
7
were randomly assigned to receive either carvedilol (a beta-blocker) or a placebo. In their report of the results
8
of the study. the investigators included the table with baseline characteristics (i.e .. age. race. prevalence of
:[lJ hypertension. etc.) of the patients in the treatment and placebo groups. According to the table. both groups
had a similar distribution of these characteristics. With the information given. which of the following is most
11 probable?
12
13 r A. The sample size is adequate [5%]
14 r B. The study is negative [4%]
15
r C. The power of the study is big [8%]
16
17 ., r D. Randomization is successful [74%]
·liBl r E. Observer's bias might be an issue [9%]
·~
• 20
·1211 Explanation: User ld:
. LBJ The purpose of randomization is to make the distribution of all potential confounders even (i.e .. between the
23
24 treatment and placebo groups). Unlike all the other methods of controlling confounding (e.g .. matching.
25 stratified analysis). randomization potentially controls known. as well as unknown confounders. One of the
26 methods to assess the adequacy of randomization is to look at the distribution of baseline characteristics in
both groups. If they are similar. one can assume that the randomization evenly distributed the confounders
·1271 between the groups. and that randomization was successful.
·~
29
(Choices A and C) From the information given. we cannot judge the adequacy of the sample size to detect
30
the difference in the survival (if it is present) between the treatment and placebo groups. We also cannot
judge the power of the study based on the above information.
32
33 (Choice B) The results of the study were not mentioned.
34
35 (Choice E) Observer's bias can be controlled with blinding. not randomization.
36
37 Educational Objective:
38 In clinical trials. randomization is said to be successful when a similarity of baseline characteristics of the
39 patients in the treatment and placebo groups is seen.
40
1.":~
• 1
• 2 Item: 32 of 44 11 PMark -<J C>- jf ""• I

:CJ 5
Q.ld : 4184 [ Previous Ne><t

A 35-year-old Caucasian female presents to the office due to a self-palpated breast mass. After the
appropriate work-up. fine-needle aspiration (FNA) of the mass is performed. The results of the FNA return as
Lab Values Notes Calculator

6 negative. As you are explaining the test result. the patient asks. "What are the chances that I really do not
7 have breast cancer?" Which of the following values best addresses this patient's question?
8
r A. Sensitivity [11 %]
:[lJ r B. Specificity [13%]
11 r C. Positive predictive value [7%]
12
13
., r D. Negative predictive value [63%]
14 r E. Validity [5%]
15
16
17 Explanation: User ld:
·liBl The negative predictive value (NPV) is defined as the probability of being free of a disease if the test result is
·~ negative. One very important thing to remember is that the NPV will varv with the pretest probability of a
• 20
disease. A patient with a high probability of having a disease will have a low NPV. and a patient with a low
·1211 probability of having a disease will have a high NPV. Specific examples are given below:
. LBJ
23 1) Breast cancer and FNA test results
24
25
A patient with a high pre-test probability for having breast cancer (e.g .. female. has first degree relatives
26
with breast cancer. greater than 40 years old) has a low NPV. A patient with a low pre-test probability of
·1271 having breast cancer (e.g .. less than 40 years old. as in this case) has a high NPV.
·~
29
30
2) HIV and ELISA test results

A patient who belongs to a high-risk group (e.g .. multiple sexual partners. admits to not using condoms.
IV drug user) has a high pre-test probability; consequently. this patient will have a low NPV. On the other
34 hand. a patient who belongs to a low-risk group (e.g .. one sexual partner who has no other sexual
35 partners. uses condoms correctly all the time. no history of IV drug use) has a low pre-test probability;
36 consequently. this patient will have a high NPV.
37
38 'Note: The prevalence of a disease is directly related to the pre-test probability of having the disease. and also
39 affects the NPV.
40
~· 1.":~
• 1
• 2 Item: 32 of 44 !il f> Mark <:J C>- jf I

:CJ 5
Q.ld : 4184 [ Previous Ne><t
negar1ve. une very 1mporranr rmng ro rememoer 1s mar rne 1\ft-'V wm varv w1rn rne preresr prooaomrv or a
disease. A patient with a high probability of having a disease will have a low NPV. and a patient with a low
Lab Values Notes Calculator

6 probability of having a disease will have a high NPV. Specific examples are given below:
7
1) Breast cancer and FNA test results
8

:[lJ A patient with a high pre-test probability for having breast cancer (e.g .. female. has first degree relatives
with breast cancer. greater than 40 years old) has a low NPV. A patient with a low pre-test probability of
11
12 having breast cancer (e.g .. less than 40 years old. as in this case) has a high NPV.
13
14 2) HIV and ELISA test results
15
16 A patient who belongs to a high-risk group (e.g .. multiple sexual partners. admits to not using condoms.
17 IV drug user) has a high pre-test probability; consequently. this patient will have a low NPV. On the other
·liBl hand. a patient who belongs to a low-risk group (e.g .. one sexual partner who has no other sexual
·~ partners. uses condoms correctly all the time. no history of IV drug use) has a low pre-test probability;
• 20 consequently. this patient will have a high NPV.
·1211
. LBJ 'Note: The prevalence of a disease is directly related to the pre-test probability of having the disease. and also
23 affects the NPV.
24
25 (Choices A and B) The sensitivity and specificity of a test are fixed values which do not vary with the pretest
26 probability of a disease. Most researchers agree that the ideal diagnostic test should have high sensitivity and
·1271 specificity. In this case. FNA also has a high sensitivity and specificity; however. the statistical parameter
·~ being described by the patient and physician was the high negative predictive value.
29
30 (Choice C) The positive predictive value follows the same concept. but applies if the test result is positive.

(Choice E) Validity represents the appropriateness of the test (i.e .. the test measures what it is supposed to
measure). It does not depend on the pretest probability of the disease.
34 Educational Objective:
35 NPV is the probability of being free of a disease if the test result is negative. Remember: the NPV will varv
36 with the pretest probability of a disease. A patient with a high probability of having a disease will have a low
37 NPV. and a patient with a low probability of having a disease will have a high NPV.
38
39
Time Spent 2 seconds Copyright© USMLEWorld .LLC. Last updated [11/1 0/2011]
40
2 Item: 33 of 44 11 f>Mark <:::1 C>- if ~· 1.":~ I

:CJ
5
Q.ld: <W19 [ Previous Ne><t

Two studies were conducted in the same population to assess the relationship between oral contraceptive
use and the risk of deep venous thrombosis (DVT). Study A demonstrated an increased risk of DVT among
Lab Values Notes Calculator

6 oral contraceptive users. with a relative risk of 2.0 and 95% confidence interval of 1.2-2 .8. Study B showed a
7 relative risk of 2.01 and 95% confidence interval of 0.8-3.1 . Which of the following statements is most likely
8 true about these 2 studies?
:[lJ r A. The result in study A is not statistically significant [6%]
11
12 r B. The result in study B is biased [9%]
13 r C. The result in study A is not accurate [3%]
14
., r D. The sample size in study B is smaller than in study A [65%]
15
16 r E. The P value in study B is less than 0.05 [16%]
17
·liBl Explanation: User ld:
·~
• 20
Statistical significance can be expressed with either P values or confidence intervals. but both are interrelated:
·1211
. LBJ 1. The P value is inversely related to the confidence interval. A P value of 0.05 corresponds to a 95%
23 confidence interval. while a P value of 0.01 is equivalent to a 99% confidence interval.
24 2. If the null value (ie. 1.0) lies outside of a given confidence interval. then the P value of the study is less
25 than the equivalent confidence interval value. Conversely. if the null value is within a given confidence
26 interval. then the P value of the study is greater than or equal to the equivalent confidence interval
·1271 value.
·~
29 95% confidence 99% confidence
30 interval interval
31
32 Null value outside
P value< 0.05 P value< 0.01
confidence interval
34
Null value inside
35 P value~ 0.05 P value~ 0.01
confidence interval
36
37
38 In a statistically significant study. the P value should be less than 0.05. This corresponds to a 95% confidence
39 interval that does not include the null value. Study A is statistically significant as the 95% confidence interval
40 rlnP.~ nnt inr.h !riP. thP. mill v;:~h IP. IChoice A\ ThP. Rl;% r.nnfir!P.nr.P. intP.rv;:~l fnr ~til rill R inr.l11r!P.~ thP. mill v;:~IIIP.
2 Item: 33 of 44 !il PMark -<J C>- jf ~ 1.":~ I

:CJ
5
Q.ld: <W19 [ Previous Ne><t
1. The P value is inversely related to the confidence interval. A P value of 0.05 corresponds to a 95%
confidence interval. while a P value of 0.01 is equivalent to a 99% confidence interval.
Lab Values Notes Calculator

6 2. If the null value (ie. 1.0) lies outside of a given confidence interval. then the P value of the study is less
7 than the equivalent confidence interval value. Conversely. if the null value is within a given confidence
8 interval. then the P value of the study is greater than or equal to the equivalent confidence interval
value.
:[lJ
11 95% confidence 99% confidence
12 interval interval
13
14 Null value outside
P value< 0.05 P value< 0.01
15 confidence interval
16 Null value inside
17 P value~ 0.05 P value~ 0.01
confidence interval
·liBl
·~
• 20 In a statistically significant study. the P value should be less than 0.05. This corresponds to a 95% confidence
·1211 interval that does not include the null value. Study A is statistically significant as the 95% confidence interval
. LBJ does not include the null value (Choice A). The 95% confidence interval for study B includes the null value .
23 and thus the study has a P value greater than 0.05 (Choice E).
24
25 The confidence interval in study B is wider than in study A. most likely due to a smaller sample size. A small
26 sample size can lead to lack of statistical significance from insufficient power to detect the difference between
exposed and unexposed subjects. Increasing the sample size would increase the studies· power and make
·1271 the confidence interval tighter. By doing so. the null value can be excluded. thus increasing statistical
·~ significance.
29
30
(Choices B & C) The accuracy or validity of the studies cannot be judged because there is no information on
31
how they were designed and conducted.
32
Educational objective:
34 The power of a study represents its ability to detect a difference between 2 groups (eg. exposed versus
35 nonexposed) when there truly is a difference. Increasing the sample size increases the power of a study and
36 consequently narrows the confidence interval surrounding the point estimate (best guess). Confidence
37 intervals express statistical significance and can be converted into P values.
38
39
Time Spent 2 seconds Copyright© USMLEWorld .LLC. Last updated [2/14/20 13]
40
2 Item: 34 of 44 1111 PMark <J C>- jf 9-• 1.~ I

:CJ
5
Q.ld: 4107 [ Previous Ne><t

Two cross-sectional studies were conducted using different questionnaires to determine the prevalence of
Lab Values Notes Calculator

6
over-the-counter analgesics use in a population. The first study showed a prevalence of 7.5% (95%
7
confidence interval 6.0 - 9 .0). and the second study demonstrated a prevalence of 7.3% (95% confidence
8
interval 6.9 - 7 .6). If the true prevalence of over-the-counter analgesics use in the population is 7.4%. which of
:[lJ the following statements about the results of the study is the most accurate?
r A. The first study results are more specific [3%]
11
12 r B. The second study results are more sensitive [9%]
13
r C. The first study results are more valid [5%]
14
15 r D. The first study results are more accurate [5%]
16 ., r E. The second study results are more precise [78%]
17
·liBl
·~ Explanation: User ld:
• 20
·1211 Precision is the measure of random error in the study. The study is precise if the results are not scattered
. LBJ widely; this is reflected by a tight confidence interval. The first study has a wider confidence interval
23 compared to the second study; therefore. the second study is more precise.
24
25 (Choices A and B) Specificity and sensitivity are measures of validity. The sensitivity and specificity of the
26
questionnaires used in these studies cannot be determined from the given information.

·1271 (Choices C and D) Validity and accuracy are measures of systematic error (bias). Accuracy is reduced if
·~ the result does not reflect the true value of the parameter measured. Increasing the sample size increases
29
the precision of the study. but does not affect accuracy. In our case. the results of both studies are pretty
30
close to the true value. and are thus seemingly accurate.
31
32 Educational Objective:
Precision is the measure of random error. The tighter the confidence interval. the more precise the result.
Increasing the sample size increases precision.

36 Last updated [7/26/20 13]


Time Spent 2 seconds Copyright© USMLEWorld .LLC.
37
38
39
40
2 Item: 35 of 44 11 PMark -<J C>- jf ""• 1.":~ I

:CJ
5
Q.ld: 4109 [ Previous Ne><t

A case-control study was conducted to assess the relationship between tampon use and toxic shock
syndrome (TSS). The odds ratio for tampon use comparing the patients with TSS and the healthy subjects
Lab Values Notes Calculator

6 (controls) was calculated and turned outto be 5.0 (p < 0 .DO 1). The authors concluded thatthe risk of TSS is
7 5 times higher in tampon users. The conclusion is valid if which of the following assumptions is satisfied?
8

:[lJ r
r
A. The sample size is big [33%]
B. The odds ratio is highly statistically significant [16%]
11
12 r C. The confidence interval for the odds ratio is tight [23%]
13
14
., r D. TSS is a rare disease [19%]
15 r E. The exposure is not associated with the disease [9%]
16
17
Explanation: User ld:
·liBl
·~ Case-control studies are very popular in exploring an exposure-disease association. because they are
• 20
relatively cheap and less time-consuming than cohort studies. One of the major drawbacks of case-control
·1211 studies is the fact that the risk cannot be derived directly from their results .
. LBJ
23 Exposure-odds ratio is the measure of association in case-control studies. It compares the odds of exposure
24 in cases to the odds of exposure in controls. It is NOT the same as relative risk. Relative risk can be
25 calculated in follow-up studies by comparing the risk in exposed individuals to the risk in unexposed
26 individuals. Direct calculation of the relative risk is not possible in case-control studies. because the study
·1271 design does not include following people over time. Nevertheless. the relative risk can sometimes be
·~ approximately equal to the odds ratio. If the prevalence of the disease is low. the exposure odds ratio
29 approximates the relative risk. This statement is called 'the rare disease assumption' and represents one of
30 the fundamental epidemiologic concepts.
31
32 (Choices A, B and C) Increasing the sample size would decrease the 'p' value for the odds ratio and make
33 the confidence interval tighter. The precision of the estimate would increase; but. that fact does not affect the
34 approximation of the odds ratio to the relative risk.

36 (Choice E) The results of the study suggest that the exposure is associated with the disease.
37
38 Educational Objective:
39 If the outcome of a case-control study is not common in the population. the odds ratio is close to the relative
40 risk.
2 Item: 36 of 44 !il PMark <::::1 t> jf ~· !:':~ I

:CJ
5
Q.ld: 2138 [ Previous Next

Researchers at a large pharmaceutical company discover a tumor-specific antigen present in high quantities
in the serum of patients with pancreatic cancer. A study is then performed to evaluate serum levels of the
Lab Values Notes Calculator

6 tumor marker in patients with and withoutthe disease. The following curves are generated using the results
7 of the study.
8

:[lJ 0 Healthy A B
11
12
13
D Diseased
14 ....<=
Ill

15 Ql

16 ·.::
17
·liBl
..."'a.....
0

·~
Ql
..c
• 20 E
:::l
·1211 z
. LBJ
23
24
25
26
·1271 Ant igen concentration
·~ ($) USMLEWorld, LLC
29
30
The drug company decides to utilize the newly discovered tumor antigen by creating a screening test for
31
pancreatic cancer. During preliminary design of the test, the cutoff point for positive/negative results is set at
32 point B. If the cutoff point is moved from B to A, the sensitivity of the screening test will change in which of the
33
following ways?

r A. Cannot be determined [0%]


r B. Decreased [15%]
38 ., r C. Increased [84%]
39
40 r D. Unchanqed fO%1 .::1
~· 1.":~
• 1
• 2 Item: 36 of 44 ill PMark <:J C>- jf I

:CJ 5
Q.ld: 2138 [ Previous

Test cutoff
Ne><t Lab Values Notes Calculator

6
7 I - - Negative test ---il 1
l-- - Positive test - - - - ;
8

:[lJ ...."'c
41
·;;;
11
12
13
..."'...c.
0
14 41
..c
15 E
16 ::l
z
17
·liBl
·~
• 20
·1211 Ant igen concentration
. LBJ © U SMLEWo~d.LLC
23
24
25 Important parameters of diagnostic tests include the following:
26
• True positive (TP)- A patient with a positive test result who actually has the disease.
·1271 • False positive (FP) - A patient with a positive test result who actually does not have the disease.
·~ • True negative (TN)- A patient with a negative test result who actually does not have the disease.
29
• False negative (FN) - A patient with a negative test result who actually has the disease.
30
• Sensitivity- The number of patients correctly testing positive for a disease divided by the total number
31
of patients with the disease (TP I [TP + FN]). High sensitivity means that negative results are less
32
likely to be FNs and more likely to be TNs; thus. a negative result is better able to rule out the disease
33
(SnOut). Because tests with high sensitivity will correctly identify most of the patients with the
disease. sensitivity is very important for screening tests (which need to minimize FNs).
• Specificity- The number of patients correctly testing negative for a disease divided by the total number
of patients without the disease (TN I [TN + FP]). High specificity means that positive results are less
likely to be FPs and more likely to be TPs; thus. a positive result is better able to rule in the disease
38
(Spin). Because tests with high specificity will correctly identify most of the patients without the
39
disease. specificity is very important for confirmatory tests (which need to minimize FPs).
40
~· 1.~
• 1
• 2 Item: 36 of 44 11 PMark -<:J C>- jf I

:CJ Q.ld: 2138 [ Previous Ne><t Lab Values Notes Calculator

5
6
7
l/ Antigen concentration "-·
8

:[lJ © U SMLEWo~d.LLC

11
12
Important parameters of diagnostic tests include the following:
13 • True positive (TP)- A patient with a positive test result who actually has the disease.
14 • False positive (FP) - A patient with a positive test result who actually does not have the disease.
15 • True negative (TN)- A patient with a negative test result who actually does not have the disease.
16 • False negative (FN) - A patient with a negative test result who actually has the disease.
17 • Sensitivity- The number of patients correctly testing positive for a disease divided by the total number
·liBl of patients with the disease (TP I [TP + FN]). High sensitivity means that negative results are less
·~ likely to be FNs and more likely to be TNs; thus. a negative result is better able to rule out the disease
• 20 (SnOut). Because tests with high sensitivity will correctly identify most of the patients with the
·1211 disease. sensitivity is very important for screening tests (which need to minimize FNs).
. LBJ • Specificity- The number of patients correctly testing negative for a disease divided by the total number
23 of patients without the disease (TN I [TN + FP]). High specificity means that positive results are less
24 likely to be FPs and more likely to be TPs; thus. a positive result is better able to rule in the disease
25 (Spin). Because tests with high specificity will correctly identify most of the patients without the
26
·1271
disease. specificity is very important for confirmatory tests (which need to minimize FPs). r
·~ In this example. moving the cutoff point from B to A will cause more patients with the disease to test positive
29
(fewer FNs). increasing the sensitivity of the test. However. as a consequence. more patients without the
30
disease will also test positive (more FPs). resulting in decreased specificity. Conversely. moving the cutoff
31
point in the other direction (eg. A to B) will increase the number of FNs and decrease the number of FPs.
32
decreasing sensitivity while increasing specificity.
33
Educational objective:
Changing the cutoff point of a quantitative diagnostic test will inversely affect its sensitivity and specificity.
Typically. lowering the cutoff value will increase sensitivity (fewer false negatives) and decrease specificity
(more false positives). Screening tests need high sensitivity. and confirmatory tests need high specificity.
38
39
Time Spent 18 seconds Copyright© USMLEWorld .LLC. Last updated [6111120 13]
40
2 Item: 37 of 44 ill PMark <j t> I r.:J-• 1.":~ I

:CJ
5
Q.ld: 4315 [ Previous Ne><t

It is hypothesized that high glucose levels during an episode of acute myocardial infarction may be associated
with higher short-term and long-term cardiovascular outcomes in non-diabetic individuals. A group of
Lab Values Notes Calculator

6
investigators studied the relationship between the blood glucose level on admission and the number of
7
episodes of significant ventricular arrhythmias during the first 24 hours after an acute myocardial infarction in
8
patients with no previous history of diabetes. The following plot was constructed.
:[lJ Blood sugar level
11
12 150
13
14 140
15 130
• •• •
16 • • •• •
17 120
• • ••
·liBl 110
••
•• •
·~ • • ••
• 20
·1211 10 20 30 40 50
. LBJ Incidence ot errhyltwnie episodes
23
24
25 Which of the following is the best statement about the study results?
26
·1271 r A. The correlation coefficient is negative [2%]
·~
29 r B. The correlation coefficient is close to 0 [3%]
30 ., r C. A linear association is demonstrated [88%]
31
r D. Confounders are effectively controlled [1 %]
32
33 r E. High blood sugar levels cause arrhythmia [4%]
34
35
36 Explanation: User ld:

Scatter plots are useful for crude analysis of data. These can demonstrate the type of association (i.e .. linear.
38
non-linear). if any is present. If a linear association is present. the correlation coefficient can be calculated to
39
provide a numerical description of the linear association. In this case. the scatter plot shows an almost
40 . . .
2 Item: 37 of 44 !II PMark <:J C>- jf ~· 1.":~ I

:CJ
5
Q.ld: 4315 [
10 20 30 40 50
Incidence ot errhyltwnie episodes
Previous Ne><t Lab Values Notes Calculator

6
7
8 Which of the following is the best statement about the study results?

:[lJ r A. The correlation coefficient is negative [2%]


11
r B. The correlation coefficient is close to 0 [3%]
12
13 ., r C. A linear association is demonstrated [88%]
14 r D. Confounders are effectively controlled [1 %]
15
r E. High blood sugar levels cause arrhythmia [4%]
16
17
·liBl Explanation: User ld:
r
·~
• 20 Scatter plots are useful for crude analysis of data. These can demonstrate the type of association (i.e .. linear.
·1211 non-linear). if any is present. If a linear association is present. the correlation coefficient can be calculated to
. LBJ provide a numerical description of the linear association. In this case. the scatter plot shows an almost
23 perfect linear association between the blood glucose level on admission and the number of episodes of
24 significant ventricular arrhythmias.
25
26 (Choice A) The scatter plot demonstrates a positive association (i.e .. an increase in the blood glucose level is
·1271 associated with an increase in the number of episodes of significant ventricular arrhythmias); therefore. the
·~ correlation coefficient is positive.
29
30 (Choice B) The correlation coefficient of an almost perfect linear association is close to 1.
31
(Choices D and E) Crude analysis of the association using scatter plots does not account for possible
32
confounders (e.g .. severity of the disease. degree of sympathetic activation. etc.). and does not necessarily
33
imply causal relationships between variables.
34
35 Educational Objective:
36 Scatter plots are useful for crude analysis of data. These can demonstrate the type of association (i.e .. linear.
non-linear). if any is present.
38
39
Time Spent 3 seconds Copyright© USMLEWorld .LLC. Last updated [7/7/20 10]
40
1.":~
• 1
• 2 Item: 38 of 44 11 PMark <:1 1>- jf 9'' I

:CJ 5
Q.ld: 3915 [ Previous Ne><t

A study was conducted to assess the relationship between ethnicity and end-stage renal disease. Two
Lab Values Notes Calculator

groups of pathologists independently studied specimens from 1.DOD kidney biopsies. The first group of
6
pathologists was aware of the race of the patient from whom the biopsy came. while the second group was
7
blinded from the patient's race. The first group reported 'hypertensive nephropathy' much more frequently for
8
black patients than the second group. Which of the following types of bias is most likely present in this study?
:[lJ r A. Confounding [3%]
11 r B. Respondent bias [4%]
12
r C. Recall bias [2%]
13
14 r D. Selection bias [5%]
15 ., r E. Observer bias [84%]
16
17
·liBl Explanation: User ld:
·~ Observer bias occurs when the investigator's decision is adversely affected by knowledge of the exposure
• 20
status. In this case. some pathologists' decisions were influenced by the fact that hypertensive nephropathy
·1211
. LBJ is a common cause of ESRD in the black population. The pathologists who were blinded from the patients'
23 race were not under this influence. so their interpretation was more unbiased.
24
25 (Choice A) Confounding is present when at least part of the exposure-disease relationship can be explained
26
by another variable (confounder). No information on possible confounders is given in this scenario.

·1271 (Choice B) Respondent bias is present when the outcome is obtained by the patient's response. and not by
·~ objective diagnostic methods (e.g .. migraine headache). In this case. the diagnosis was ascertained via
29
kidney biopsy.
30
31 (Choice C) Recall bias results from inaccurate recall of past exposure by patients. It is not applicable to this
32 case.
33
34 (Choice D) Selection bias results from the manner in which the subjects are selected for the study. or from
35 the selective losses from the follow-up.
36
Educational objective:
Observer bias occurs when the investigator's decision is adversely affected by knowledge of the exposure
status.
2 Item: 39 of 44 11 PMark <J C>- jf 9-• 1.~ I

:CJ
5
Q.ld: <W02 [ Previous Ne><t

Ten measurements of systolic blood pressure were obtained from a patient in the ICU using an intra-arterial
Lab Values Notes Calculator

6 cannula over several hours. The maximal value recorded is 120 mmHg. and the minimal value is 1DO mmHg.
7 If the next measurement shows the value of 240 mmHg. which of the following is most likely to happen?
8

:[lJ ., r
r
A. The mean would increase significantly [80%]
B. The median would increase significantly [1 0%]
11
12 r C. The mode would increase significantly [4%]
13 r D. The standard deviation would not change [4%]
14
15
r E. The range would not change [1 %]
16
17 Explanation: User ld:
·liBl
·~ An outlier is defined as an extreme and unusual value observed in a dataset. It may be the result of a
• 20 recording error. a measurement error. or a natural phenomenon. An outlier can affectthe measures of central
·1211 tendency. as well as the measures of dispersion. For example. the mean is extremely sensitive to outliers
. LBJ and easily shifts toward them. In this case. the value of 240 mmHg is the outlier.
23
24 (Choice B) The median is much more resistant to outliers. because it is located in the middle of the dataset
25 where the observations usually do not differ much from each other.
26
·1271 (Choice C) The mode is not affected by outliers. because they do not change the most frequent value
·~ observed.
29
30 (Choice D) The standard deviation is sensitive to outliers. because it is the measure of dispersion within the
31 dataset. and outliers significantly increase the dispersion.
32
(Choice E) The range is equal to the maximal value minus the minimal value; therefore. it would definitely
33
change.
34
35 Educational objective:
36 An outlier is defined as an extreme and unusual observed in a dataset. The mean is very sensitive to outliers
37 and easily shifts toward them. The median and mode are more resistant to outliers.
38

Time Spent 2 seconds Copyright© USMLEWorld .LLC. Last updated [7/26/20 13]
2 Item: 40 of 44 11 PMark -<J C>- jf ""• 1.":~ I

:CJ
5
Q.ld: 4172 [ Previous Ne><t

In a large population with little migration. the incidence of diabetes mellitus. type II is equal to 3 cases per
1.000 per year. and has been stable for the last 30 years. The prevalence of this disease increased
Lab Values Notes Calculator

6 progressively over the same period. Which of the following could be the most likely explanation of this trend
7 overtime?
8

:[lJ r
r
A. High mortality in diabetics [2%]
B. Selective survival [4%]
11
12 ., r C. Improved quality of care [84%]
13
r D. Decreased hospitalization rate [2%]
14
15 r E. Increased diagnostic accuracy [8%]
16
17
Explanation: User ld:
·liBl
·~ Incidence and prevalence are two important concepts in epidemiology. It is thus very important to know the
• 20 difference between these two. Incidence is the measure of the appearance of new cases. Prevalence is the
·1211 measure of those with the disease in the population at a particular point in time. The relationship between
. LBJ these two categories can be demonstrated by the following approximation in a stable population (little
23 migration)
24
25
26
Prevalence =(Incidence) x (Time)
·1271 The above vignette described a disease with a rising prevalence but stable incidence. More people are being
·~ documented to have the disease. while the number of people being diagnosed remains the same. Such trend
29
can be attributed to factors which prolong the duration of the disease (e.g .. improved quality of care).
30
31
(Choice A) A high mortality rate in diabetics would result in a decreased prevalence.
32
33 (Choices B and D) Selective survival and decreased hospitalization rate are not helpful in explaining the
34 increased prevalence described in this scenario.
35
36 (Choice E) Increased diagnostic accuracy affects both the prevalence and incidence of a disease.
37
38 Educational Objective:
An increasing prevalence and stable incidence can be attributed to factors which prolong the duration of a
disease ( e .a .. imoroved aualitv of care - this scenario is tvoical for the USMLEl.
.
~· 1.~
1 •
. Item: 41 of 44 11 PMark <:J C>- jf
·w
2 I

Q.ld: 3992 [ Previous Ne><t Lab Values Notes Calculator


• 4
5
6 A group of investigators are planning a study to evaluate the relationship between serum fibrinogen level and
7 the incidence of acute coronary syndrome. They assume that serum fibrinogen level is a normally distributed
8 variable in the population of interest. Which of the following statements is most consistent with this
·[;J
• 10
assumption?

11
12 r A. Mean is greater than median [5%]
13 r B. Mean is greater than mode [3%]
14 ., r C. Mean is equal to median [85%]
15
16 r D. Median is greater than mean [3%]
17 r E. Mode is greater than mean [3%]

·[m
• 19
. 20 Explanation: User ld:

•-~
22 The normal distribution is one of the most popular statistical distributions. Interestingly. many variables in real
23 life (e.g .. laboratory values) are normally distributed or close to normal. The normal distribution has some nice
24 statistical properties. and is easy to work with. It is bell-shaped and symmetric. Consequently. all its
25 measures of central tendency are equal: mean = median = mode (In real life. the values are very close to
26 each other). Skewed distributions do not have this property.

·c:]
• 28
(Choices A and B) In a positively skewed distribution (tail on the right). the mean is greater than the median
and greater than the mode.
29
30 (Chocies D and E) In a negatively skewed distribution (tail on the left). the mean is less than the median and
31 less than the mode.
32
33 Educational Objective:
34 A normal distribution is symmetric and bell-shaped. All its measures of central tendency are equal: mean=
35 median= mode.
36
37 Time Spent 2 seconds Copyright© USMLEWorld .LLC. Last updated [7/7/20 10]
38
39
40
.
~· 1.":~

.
1
Item: 42 of 44 !il PMark <J C> II
·w
2 I

Q.ld: 3909 [ Previous Ne><t Lab Values Notes Calculator


• 4 A prospective cohort study was conducted to evaluate the long-term effects of a high-fat diet on the incidence
5 of colon cancer. The study participants were randomly selected from the population of interest. Dietary
6 patterns were assessed through the use of periodic self-completed questionnaires. The investigators
7 reported a 5-year relative risk of 1.60 for people who consumed a high-fat diet compared to individuals who
8 consumed a low-fat diet. The 95% confidence interval was 1.02 to 2.15. This study is most likely to have

·[;J
• 10
which of the following p values?

11 .., r A. 0.04 [81 %]


12
13 r B. 0.06 [7%]
14 r c. 0.09 [5%]
15 r D. 0.11 [3%]
16
17 r E. 0.20 [3%]

·[m
• 19
. 20 Explanation: User ld:

•-~
Relative risk (RR) is the probability of the outcome of interest occurring in the exposed group compared to the
22 probability of it occurring in the non-exposed group. The null value of the RR is 1.0; a RR of 1.0 means that
23 the outcome occurs with equal frequency in both groups and that there is no association between the
24 exposure and the outcome. A RR > 1.0 means that the outcome occurs more frequently in the exposed
25 group (positive association). The RR says nothing about the statistical significance of a study.
26

·c:]
• 28
Statistical significance can be expressed with either p values or confidence intervals. but both are
interrelated. For instance. p < 0.05 corresponds to a 95% confidence interval that does not contain the null
29 value. Likewise p < 0.01 is equivalent to a 99% confidence interval that does not contain the null
30 value. Conversely. if the null value is within a given confidence interval. then the p value is~ the equivalent
31 confidence interval.
32
33 95% confidence 99% confidence
34
35 interval interval
36
37 Null value outside
38 p<O.OS p < 0.01
39 confidence intervals
40
.
~· 1.":~
1 •
. Item: 42 of 44 !il PMark <:J C>- jf
·w
2 I

Q.ld: 3909 [ Previous Ne><t Lab Values Notes Calculator


• 4 ' ' " '-' '
exposure and the outcome. A RR > 1.0 means thatthe outcome occurs more frequently in the exposed
5 group (positive association). The RR says nothing about the statistical significance of a study.
6
7 Statistical significance can be expressed with either p values or confidence intervals. but both are
8 interrelated. For instance. p < 0.05 corresponds to a 95% confidence interval that does not contain the null
·[;J
• 10
value. Likewise p < 0.01 is equivalent to a 99% confidence interval that does not contain the null
value. Conversely. if the null value is within a given confidence interval. then the p value is~ the equivalent
11 confidence interval.
12
13
95o/o confidence 99% confidence
14
15 interval interval
16
17 Null value outside
·[m
• 19 confidence intervals
p<O.OS p < 0.01
. 20

•- ~
Null value inside
22 p ~ 0.05 p~O.O l
23 confidence interval
24
@ USMI£World. l l C
25
26
In general. for study results to be statistically significant. the 95% confidence interval must not contain the null
·c:]
• 28 value. This is equivalent to p < 0.05. which means that there is less than a 5% chance that the results are
due to chance alone. In this case. the 95% confidence interval is 1.02 to 2.15 and does not contain the null
29
30 value of 1.0. This corresponds to p < 0.05. thus 0.04 is the only correct answer choice.
31
(Choices 8, C, D, and E) These p values are all> 0.05. meaning the associated 95% confidence interval
32
would contain the null value.
33
34 Educational objective:
35 Confidence intervals and p values are interrelated and express the statistical significance of a study. In a
36 statistically significant study. p should be< 0.05. This corresponds to a 95% confidence interval that does not
37 include the null value.
38
39 Last updated [7121/20 13]
Time Spent 2 seconds Copyright© USMLEWorld .LLC.
40
.
~· 1.":~
1 •
. Item: 43 of 44 11 PMark -<:1 C>- jf
·w
2 I

Q.ld: 3982 [ Previous Ne><t Lab Values Notes Calculator


• 4
5
6 Consecutive readings of pulmonary capillary wedge pressure (PCWP) were obtained from a patient in the
7 Intensive Care Unit (ICU) using a Swan-Ganz catheter. The readings are 20 mmHg. 22 mmHg. 21 mmHg. 22
8 mmHg. and 18 mmHg. Which of the following is the median of the values given above?
·[;J
• 10
r A. 18 mmHg [2%]
11
12 r B. 20 mmHg [8%]
13 .., r C. 21 mmHg [79%]
14
15
r D. 22 mmHg [5%]
16 r E. 20.6 mmHg [6%]
17

·[m
• 19 Explanation: User ld:
. 20
It is important to know the difference between the measures of central tendency. The median of a dataset is
•-~
22 the number that divides the right half of the data from the left half. In this case. 21 mmHg is in the middle of
23 the dataset; therefore. it is the median. If the number of observations is even. finding the median becomes
24 tricky. You should find the middle two values. add them together. and divide by two.
25
(Choices A and B) are not measures of the center in this dataset.
26

·c:]
• 28
(Choice E) To find the mean of a dataset. you should add all the observations and divide that sum by the
number of observations. In this case. the mean is equal to 20.6 mmHg.
29
30 (Choice D) Another measure of the center of a dataset is the mode. Finding the mode is the easiest. The
31 mode is the most frequent value of a dataset. In the scenario described. the mode is 22 mmHg.
32
33 Educational Objective:
34 The median is the value that is located in the middle of a dataset. It divides the right half of the data from the
35 left half.
36
37 Time Spent 2 seconds Copyright© USMLEWorld .LLC. Last updated [7/7/20 10]
38
39
40
~· 1.~~

Item: 44 of 44 11 PMark <:::1 1>- jf
·w
2 I

Q.ld: 3941 [ Previous Ne><t Lab Values Notes Calculator


• 4 A large clinical trial is performed to evaluate the treatment response to statins in statin-naive patients admitted
5 for anginal chest pain. Patients are randomly assigned to moderate- or high-dose statin therapy upon
6 discharge. At 3 months. the adverse cardiovascular event rate in the high-dose group compared to the
7 low-dose group was 0.98 with a p value of 0.80. At 1 year. the relative risk ratio was 0.67 with a p value of
8 0.01. Treatment and follow-up compliance was high in both groups throughout the study interval. The
·[;J
• 10
difference between the 2 risk estimates is best explained by which of the following?

11
12
r A. Confounding bias [5%]
13 ., r B. Latency period [65%]
14 r C. Observer bias [3%]
15
16
r D. Selection bias [4%]
17 r E. Selective survival bias [23%]

·[m
• 19
. 20 Explanation: User ld:

•-~
Latency is an important natural phenomenon of disease epidemiology. Most infectious diseases have
22
relatively short latency periods (ie. the time elapsed from initial exposure to clinically apparent disease). In
23
contrast. some disease processes (eg. cancer. heart disease) have a long latency period before clinical
24
manifestations develop.
25
26 The concept of a latency period can also be extended to risk factors and risk reducers. Sometimes. a
·c:]
• 28
significant amount of time must pass before exposure to a risk modifier has a clinically evident effect on the
disease process. In addition. exposure to a risk modifier may need to occur continuously over a certain period
29 before the disease outcome is affected. In this case. at least 1 year of high-dose statin therapy was required
30 to show a significant protective advantage over moderate-dose therapy.
31
32 (Choice A) Confounding bias occurs when a perceived exposure-disease relationship is actually caused by
33 an extraneous factor that correlates with both the exposure and the disease. There is no information
34 suggesting that confounding occurred in this study.
35
36 (Choice C) Observer bias occurs when an observer misclassifies data due to individual differences in
37 interpretation or preconceived expectations regarding the study. It can be reduced by performing a
38 double-blind study with multiple observers to encode and verify recorded data.
39
40 (Choice D) Selection bias can occur with inappropriate (ie. nonrandom) assiqnment methods or throuqh
~· 1.":~

Item: 44 of 44 !il PMark <::1 C>- jf
·w
2 I

Q.ld: 3941 [ Previous Ne><t Lab Values Notes Calculator


• 4
Explanation: User ld:
5
6 Latency is an important natural phenomenon of disease epidemiology. Most infectious diseases have
7 relatively short latency periods (ie. the time elapsed from initial exposure to clinically apparent disease). In
8 contrast. some disease processes (eg. cancer. heart disease) have a long latency period before clinical
·[;J
• 10
manifestations develop.

11 The concept of a latency period can also be extended to risk factors and risk reducers. Sometimes. a
12 significant amount of time must pass before exposure to a risk modifier has a clinically evident effect on the
13 disease process. In addition. exposure to a risk modifier may need to occur continuously over a certain period
14 before the disease outcome is affected. In this case. at least 1 year of high-dose statin therapy was required
15 to show a significant protective advantage over moderate-dose therapy.
16
17 (Choice A) Confounding bias occurs when a perceived exposure-disease relationship is actually caused by

·[m
• 19
an extraneous factor that correlates with both the exposure and the disease. There is no information
suggesting that confounding occurred in this study.
. 20
•-~
(Choice C) Observer bias occurs when an observer misclassifies data due to individual differences in
22 interpretation or preconceived expectations regarding the study. It can be reduced by performing a
23 double-blind study with multiple observers to encode and verify recorded data.
24
25 (Choice D) Selection bias can occur with inappropriate (ie. nonrandom) assignment methods or through
26 selective attrition of the study participants. It results in a study population that does not accurately represent
the actual population. leading to erroneous conclusions regarding the exposure-disease relationship.
·c:]
• 28
(Choice E) Selective survival bias occurs in case-control studies when cases are selected from the entire
29
disease population instead of just those that are newly diagnosed. For instance. a study on cancer survival
30
that is not limited to newly diagnosed patients will contain a higher proportion of relatively benign malignancies
31
as these patients generally live longer.
32
33 Educational objective:
34 The concept of a latency period can be applied to both disease pathogenesis and exposure to risk modifiers.
35 Exposure to risk factors and the initial steps in disease pathogenesis sometimes occur years before clinical
36 manifestations are evident. In addition. exposure to risk modifiers may need to be continuous over a certain
37 period before influencing the outcome.
38
39 Last updated [7/11/20 13]
Time Spent 5 seconds Copyright© USMLEWorld .LLC.
40

Você também pode gostar